Sie sind auf Seite 1von 54

* CASE 79 anoresia.

and weight loss Clinical Presentation A 28 year-old man complaits of epigastric pain
g.79.1 (A.B) Aial cotrast-enhanced CTimages of the abdomen show mild enhanicement and diffuse
increased thid neis of the gastsic walls these findings are sunpicious for gastric be escluded Mutiple
although an inflammatory process cannot neoplasm hypodeme cystic lesions are seen in the liver and
both kidneys Radiologic Findings demonstrate Axial contrast-enhanced computed tomography (CT)
images of the abdomen (Fig. 79.1) mild and diffuse thickening of the gastric Fig. 79.2) demonstrate
abnormal and persistent walls. The images from the upper gastrointestinal (GI) series narrowing of the
distal stomach. Diagnosis Gastric adenocarcinoma Differential Diagnosis Gastric metastases Gastritis
Gastric mucosa-associated lymphoid tissue lymphoma Gastric Crohn disease Caustic ingestion Gastric
tuberculosis Discussion Background Gastric carcinoma is a deadly disease, with decreasing incidence in
the past 50 years in North and Western Europe owing to the earlier incidence of gastrnc carcinoma is
extremely variable worldwide, with the higheomber of cases America identification at imaging and
endoscopy. Currently. the 250

2 TO Fg 792 (AR) e ga mate an aemally ne Ne dete cera s stial se imaon to ad dt e ont ted nt ba tet for g
m Japan. Chira Rusia and Eastern Europe (30 cases per 100.000) men are usually 2 times more commonly
allected Different than women, with a peak incidence in those ages of 50 to 70 years intestinal and
diffuse histologic types have been described adenocarcinomas types remaining 5% Adenocarcinomas
have been ct eomyosarcomas, carcinoids, and sarcomas for the papillary, tubular malignancies anse from
the antrum 30% from the account for almost 951 of cases and lymphomas into four different subrypes
mucinous and signet ing Almost 30% of gastric body, and 40% from the fundus and cardia region Clinical
Findings (early Chinical presentation mostly stage) or may present with bleeding (hematemesis depends
on the disease stage: epigastric pain, melena, occult blood in stools) patients may be asymptumatic
anorexia, weight loss, nausea, vomiting, and upper G Complications stages, liver metastases include the
lung. metastases (Krukenberg In advanced metastases may cause hepatomegaly and jaundice; less
common sites for adrenal glands, and kidneys. Pelvic masses can result from ovarian tumors) Etiology
identified: dietary nitrates, certain f resulting from drugs or previous surgery Several dietary and
environmental factors have been foods (salted fish and meat, smoked foods), hypochlorhydria cigarette
infection. In particular, patients with H. pylori gastritis are 3 to 6 smoking, and Helicabacter pylori times
more likely to develop gastric cancer. Other gastric polyps, partial gastrectomy precancerous conditions
include Ménétrier disease and pernicious adenomatous anemia

Endoscopy is currently considered the diagnostic procedure of first choice in patients with sos studies,
either with single or double contrast. have been performed extensively for the detection of gastric
tumors, although their overall sensitivity is somewhat lower than endoscopy 252 TEACHING ATLAS OF
ARDOMINAL MAGING Imaging Findings pected gastric malignancies nodes (N) and distant Conventional
CT has beep limited generally to the identification of lymph metastases (M) The recent introduction its
multiplanar capabilities, has dramatically improved the detection of primary nancies (T): moreover and
therefore of intramural invasion. Early gastric neoplasms Ing in the lumen) superficial (associated only
with irregularity of gastric (malignant ulcer) features. rigidity, and narrowing of the stomach fat
stranding. Mucin-producing gastric and a predominantly low-attenuation value corresponding to the
mucin pool thickened middle and outer layer. CT is also useful in defining the N stage: system is based on
the number of regional nodes involved in the perigastric group and the celiac axis: Gastric Barium f
multidetector CT. with its thin-slice collimation and gastric malig- the water-filling method allows a
detailed visualization of the gastric wall may present as polypoid (protrud- (olds) and excavated I cm at
CT) diffuse Advanced tumors cause thickening of the walls ( perigastric (linitis plastica) and may be
associated with features including multiple calcifications tumors show located within the the TNM
classification around (gastrohepatic and hepatis. hepa- lymph nodes (found in the mesentery considered
as distant metastases. Ascites represent a common cancer initially tends to spread to local lymph nodes
gastrocolic lymph nodes) and then to regional lymph nodes located in the porta toduodenal ligament.
and peripancreatic region. or retroperitoneum) are usually finding in patients with diffuse High spatial
resolution magnetic resonance imaging, used in in vitro experimental studies, is a promising imaging
modality for the assessment of the depth of gastric tumor invasion of its accuracy in depicting the
Distant peritoneal carcinomatosis because layers of the gastric wall. Treatment Complete ment, although
a surgical procedure may be associated with several late complications, including dumping syndrome,
vitamin B deficiency, and reflux benefit from endoscopic mucosal resection. Chemotherapy is
administered in patients with systemic metastatic disease resection of the tumor and adjacent lymph
nodes is considered the only curative treat- esophagitis; some selected neoplasms may Prognosis Gastric
cancer gastric cancers are potentially Hence prompt diagnosis of these tumors is mandatory presents a
dismal prognosis, with overall 5-year survival early rates < 20%; however, curable, with a mean 5-year
survival rate of almost 90% PEARL Distention of the gastric lumen with a negative contrast m detection of
small superficial gastrice lesions , air) may improve the

*CASE 80 Clinical Presentation A year old man peents with abuluminal pain shows a lotuate Fig. 80.1
Contrast-enhanced CT scan heteogeneously enhancing mass arising from the proial itomach (arrow)
Radiologic Findings An abdominal computed tomography (CT) scan (Fig. 80.1) shows a large, lobulated,
heterogeneously enhancing mass arising from the proximal stomach. Diagnosis Gastrointestinal stromal
tumor (GIST) Differential Diagnosis Gastric adenocarcinoma Leiomyoma/leiomyosarcoma
Schwannoma/neurofibroma Metastasis Discussion Background GISTS are the most common
mesenchymal tumags arising from the gastrointestinal (GI) tract Nevertheless, they are uncommon,
accounting far only 1 to 3% of all GI neoplasms, and usually patients 50 years of age without any gender
predilection. These tumors are also reported to occur in 10 to 25% of patients affected by
neurofibromatosis L Previously neural tumors, GISTS show distinctive genetic and histologic features. Thi
can the rectum to the esophagus, the stomach being the most comm located in the mesentery.
omentum, and retroperitoneum. wall. affect mistaken for smooth muscle and cur anywhere from on:
they can also be 254

255 vw GITRACT PERIIONE AL CAVETY, AND RETROPERITONEUM Clinical Findings clinical presentation
relies on the location, size, and pattern of growth of the tumor and is mainly zed by abdominal pain or
distention The charas obstruction. anemia or melena, or bowel Complications and less Malignant GIST
can be aggressive and spread to the liver, retroperitoneum, and mesentery, a bowel obstruction
frequently to the bones, hungs, and lymph nodes Intraluminal GISTS can cause Etiology GISTS have
distinctive immunolhistochemical (CD117), a tyosine kinase growth factor neoplastic cells the expression
of important being promotes the growth of mesenchymal properties, the most KIT receptor, which
Imaging Findings Although tumor expression of the KIT receptor is needed for a definitive diagnosis of
GIST, imaging especially CT, has proved to be instrumental. GISTS are usually enhancing, rarely calcified
masses arising from the GI tract wall. show heterogeneous density due to the presence of intratumoral
necrasis foci, and cystic seen as exophytic, heterogeneously larger ones. (Fig. 80.2) hemorrhagic Most of
them, especially degeneration Treatment Surgical removal is recommended, but despite complete
resection. the recutrence rate is high, especially in the setting of less well differentiated New drugs used
only for unresectable tumors lesions targeting the growth factor receptor KIT appear promising , but so
far they have been Prognosis Recent advancements in therapy have nonmetastatic GIST increased the
average survival rate especially in malignant 80.2 Contrast-enhanced CT in a different patient show Fig a
lurge heterogeneous mass arising from the stomach Multiple hepatic cysts are present

256 TEACHING ATLAS OF ARDOMINAL IMACING lesions should always include GISTS, especially in the
Differential diagnosis of gastrointestinal setting of an exophytic, necrotic-appearing mass associated with
the hollow viscera PEAR .GIST can be difficult to distinguish from other mesenchymal malignancies, such
as soft tissue tumors (eg, leiomyosarcoma) and neural cell tumors PITEALL Suggested Readings Burkill G),
Badran M. Al-Muderis 0, et al. Malignant imaging features, and Da Ronch T, Modesto A. Bazocchi M.
Gastrointestinal stromal tumour: spiral computed tomography teatures and pathologic correlation.
Radiol Med (Torino) 2006:111(5):661-673 Lee CM, Chen HC Leung TK, Chen YY. Gastrointestinal stromal
tumor: computed tomographic fea. tures World J Gastroenterol 2004:10( 16):2417-2418 Levy AD, Patel
N, Abbott RM, Dow N, Miettinen M, Sobin LH. Gastrointestinal stromal tumors in patients with
neurofibromatosis: 2004:183(6):1629-1636 tumor: distribution gastrointestinal stromal pattern of
metastatic spread. Radiology 2003:226:527-532 : imaging features with clinicopathologic correlation. Am
J Roentgenol Levy AD, Remotti HE, Thompson WM, Sobin LH, Miettinen M. Gastrointestinal stromal
tumors: radio- logic features with pathologic correlation. Radiographics 2003:23:283-304 Logrono R,
Bhanot P.Chava C, Cao L Waxman I, Bhutani MS. Imaging, morphologic, tochemical correlation in
gastrointestinal stromal tumors. Cancer 2006:108(4):257-266 Sandrasegaran K, Rajesh A, Rydberg J.
Rushing DA, Akisik FM, Henley JD. tumors: clinical, radiologic, and pathologic features. AJR Am J
Roentgenoal 2005;184(3):803-811 and immunohis Gastrointestinal stromal

*CASE 81 Clinical Presentation A middle aged man presents for evaluation of long standing dysphagia
with episodic perods f more intense discomfort Fig. 81.1 (A) Contrast enhanced axial CT image onstrates
a large hiatal hernia The distal esophagus is midline, and the gastroesophageal (GE) junction as well as a
portion of the stomach is located within the intratho cack hernia. (B) Overhead a scopic irmage from an
upper gastrointestinal series show a large hiatal hernia. The radiograph illustrates that the "concave
greater cephalad to the "convex" lesser curvature, the GE junction and pylorus normal relationship dem
radiograph and (C) spot fluoro- curvature of the gastric body n located The GE junction and pylorus
maintain their Radiologic Findings Contrast-enhanced axial computed tomography (CT) image
demonstrates a large hiatal 811A) The distal esophagus the stomach is located within (Fig. 81.1B,C) from
an upper gastrointestinal hernia (Fig junction as well as a portion of fluoroscapic midline, and the
gastroesophageal (GE) the intrathoracic hernia. An averhead radiograph and spot image GI) series show
a large hiatal hernia. The radiograph 257

thstrates that the "concave greater curvatue of the pastric body n located cephalad to the "coves Arss
curvatore, the GE junction and pylorus The GE punction and pylorus maintain s rionshin which exchudrs
a mesenteroacial gastric vovulux There ts no gastric outlet obstruction their norma ra 258 EACHING
ATLAS OF ARpOMNAL MACING Diagnosis Organoavial gastric volvulus in a large hiatal hernia Differential
Diagnosis Sliding Paraesophageal hernia Large intra-abdominal mass with superior displacement of the
stomach thiatal hernia Discussion Castric volvalus represents abnormal torsion of the stomnach, usually
> 180 degrees. If the greater stomach, this is termed an Background curvature rotates superior to the
lesser curvature along the long axis of the organoaxial volvulus and accounts for -60% of gastric
anterosuperior to the fundus and GE junction, this is termed a mesenteroaxial volvulus, representing
-30% of volvulus cases. The volvulus is also classified matic (two thirds) ar
subdiaphragmatic/intraperitoneal (one third) cases. If the antrum and pylorus rotate volvulus i with
respect to the diaphragm: supradiaphrag Clinical Findings present with nonspecific abdominal
complaints, including dysphagia, or dyspnea. Chronic volvulus may be noted incidentally on chest
radiographs or an upper GI series or esophagram. Acute volvulus demon- presentations, including severe
pain, an inability to ingest liquids or solids or pass there is gastric ischemia or perforation obstructing
volvulus the classic Borchardt triad describes severe constant In cases of chronic volvulus, patients
typically vague abdominal or chest pain early satiety or fullness strates more dramatic an enteric tube,
gastric outlet obstruction, and peritonitis when Purportedly diagnostic of acute chest or epigastric pain,
nausea with inability to vomit, and difficulty passing a nasogastric tube Complications Organoaxial
volvulus can be complicated by strangulation in-5 to 20% of cases secondary to twisting of the left gastric
artery and occlusion of venous drainage. Ischemia in mesenteroaxial volvulus is rare. Any type of gastric
volvulus can be associated with gastric outlet obstruction. Obstruction with strangulation is usually seen
only in cases of acute volvulus. Mortality ranges from < 10% for chronic cases to > 40% for acute volvulus
Etiology Organoaxial volvulus is usually seen in adults and is associated with a diaphragmatic defect in
50% of cases, including hiatal hernia, paraesophageal hernia, eventration, and diaphragmatc upture. This
defect allows for abnormal movement of the gastric body into the thorax with resultant o this way, the
majority of organoaxial volvulus cases also represent the supradiap tic variety oaxial volvulus In

CAVITY AND EE TePRITONEUM 259 VE GTRACT PERITONEA mesenteroaxial velvuli are overwhelmingly
subdiaphragmaric and mot often seen in children. Thoucht to be due to abmormal laxiry of the
suspensory ligaments of the stomach, mes enteroaxial volvuhus can also be associated with a
wandering" ectopic spleen In comparison Imaging Findings respect to the level consistent Plain
tadiographic stomach itself with the frequently associated .The upper volvulus, the greater curvature
findings of organoaxial Radiography alone may reveal a mediastinal volvulus are usually nonspecific with
density or air-fluid hiatal or paraesophageal hermia GI series is 1he gold standard for volvulus of
organoaxial long axis of the stomach. The GE junction may be intrathoracic or below the diaphragm
depending on the type diagnosis. In the setting curvature along the fotates superior to the lesser of
herniation. In mesenteroaxial volvulus, the pylorus rotate anterosuperior to the antrum and fundus with
a twisting around the short axis of the stomach .Cross-sectional imaging with CT and magnetic resonance
(MR) clearly demonstrates the hiatal or paraesophageal hernia with the intrathoracic stomach, Often,
the tWisting" component of the volvulus will be overlooked on axial images only: a swirling of
mesenteric vessels may be noted. -dimensional reformations for CT have improved Multiplanar MR and
three- the characterization of volvulus type with these modalities Treatment Acute gastric volvulus is
initially treated with Surgical repair return of the and repair laparotomy, or peritoneal cases with
obstruction, incarceration, or strangulation require subdiaphragmatic with endoscopic decompression
and gastropexy. reserving surgery for endoscopic attempted nasogastric tube decompression. involves
untwisting of the volvulus position, gastropexy in an anatomical position Surgical access may be via
thoracotomy volvulus can be treated electively, whereas acute repair Uncomplicated defect can be
treated is indicated for all cases of volvulus and stomach to a subdiaphragmatic of any predisposing
diaphragmatic defect laparoscopy. Chronic immediate chronic gastric volvulus cases without
diaphragmatic failure Prognosis Following related to failure of gastropexy and adequacy of repair of any
lknown diaphragmatic successful repair with gastropexy, the risk of recurrent gastric volvulus is low and
is defect PEARL Acute gastric valvulus is a surgical emergency because of the moderate risk of ischemic
stran- gulation and gastric outlet obstruction and requires diaphragmatic repair of both the gastric
torsion and any defect PITFALLS Chronic volvulus can be intermittent, and the upper GI series may only
demonstrate hiatal or araesophageal hernia during an asymptomatic period

26 1 noAL ATY AND ETRO WarET Fig 82.2 pper gastrointestinal series inage in a dif t ptit frum the medal
wall of the secnnd and thind parts of the duodenum Fig 82.3 Aial condraehced CT image i a 63- old man
shows the prence of agilled d showes the presece of diverticda ariing ing lateral wall of the thind part f
the du from the lamows) demum aw) 12 weighted magnetic resonance images show a hyperintense
tluid containing hypointense gas structure and within the nondependent portion of the lesion Treatment
Conservative treatment is preferred in asymptomatic primary duodenal diverticular disease with- out
any complications. The patient can be advised to adopt a position that favors diverticular emptying in
patients with patients with secondary disease because of an ulcer. the treatment of the ulcer by
medications and surgery is recommended Surgical treatment in symptomatic primary diverticular
disease is undertaken only complications and persistent synmptoms despite conservative management
Prognosis The prognosis for primary duodenal diverticular disease is excellent with lesser incidence of
complications. In patients with secondary diverticula the prognosis is good: however, the rates of
complications are higher due to a postbulbar ulcer who are treated PLARL The presence of gas and
continuity with the duodenum help to differentiate duodenal diverticula from other pathologies PITFAL
Dodenal diverticula filled with fluid can be mistaken for cystic neoplasms from the head of the in wihich
case barium examination should be considered

from the medal wall of the scond and third pacts ot the duotenum within 2 cm of the a Vat Intraluminal
dundenal divecticula are sare Deticua a Large in size and ane Iess cummn iv ula can alo be classified as
pimary o secondary (due to postbullar uloy ampula o tend to be Lateral wall 262 aning rom the Clinical
Findings Asympromatic Abdomnal discomfort Pain in the epigastric ar umbilical region Central abdominal
pain radiating to the hack Jaundice in the case of compression of the biliaty Pain and fever in cases of
diverticulitis tree Complications Diverticulitis erforation Pancreatitis secondary to perforation Bleeding
Obstructive jaundice due to compression of the biliary Cholangitis Failure of endoscopic retrograde the
diverticula : Increased incidence of cholelithiasis Neoplastic change channels bile duct drains into
cholangiopancreatography if the common Bezoar formation Etiology Primary diverticula are thought to
arise because of the presence of a weakened wall in the duodenum at the vessel's entry site. biliary
ductal entry sites, muscular weakness in the wall, or heterotopic pancreatic tissue Imaging Findings . On
an upper gastrointestinal series arising from the duodenum without any distortion of mucosal folds,
which changes their configu- ration with time (Fig. 82.2) Delayed retention of barium in the diverticula is
a diagnostic finding Intraluminal diverticula appear as intraluminal projections of barium-filled linear
structures with surounding lucency, giving a typical "wind Ultrasound demonstrates a clear fluid- or food-
containing structure with the presence of gas in the nondependent portion of the lesian. The wall of the
duodenal the multilayered gut signature on ultrasound. Use of real-time ultrasound with barium the
diverticula are seen as typical outpouchings sock appearance diverticula diverticula CT shows gas-
containing diverticula confused with cystic neoplasms, pseudocysts, or abscess In such onstrate the
presence of gas within the structure. typically shows imaging helps, and (Fig. 82.3) In cases where gas s
absent, diverticula can be can be better demonstrated after the ingestion of large quantities of fluids
cases, repest scans may dem-

CASE 83 Clinical Presentation A 57-year-old man presents with abdominal pain and discomfort. Physical
examination reveals a mass in the anterior palpable abdominal wall, in the midabdomen. Fig. 83.1 rim of
soft tissue density (A.B) Axlal CT image show small bowel loops hemating through the abdominal wall
and sumounded by a with stranding, consistent with mild inflammation There is mild distention af the
small bowel rOximally to the hermiated loops Radiologic Findings Axial computed tomography (CT)
images (Figs. 83.1) show abdominal wall and surrounded by a rim of soft tissue inflammation There is
mild distention of the small bowel proximally to the herniated loops through the small bowel loops
herniating with stranding consistent with mild density Diagnosis Abdominal wall hernia Differential
Diagnosis Abdominal wall tumors (desmoid, Rectus sheath hematomas (posttraumatic, associated with
Enlarged lymph nodes (infections, lymphomas, etc) Enlarged, prominent vessels (as seen in cirrhotic
patients with Undescended testis Abdominal wall endometriosis lymphoma hemangiomas, lipoma,
metastases, etc.) coagulopathy) portal venous shunts) 265

MAGING 266 TEACHING ATLAS OF ABDOMINAL Fig. 83.2 An anial contrast-enhanced CT image thiough
the pelvis demonstrates a direct left inguinal hemia Smal bowel loops are present along the left
spermatic cord Neither inflamnatory changes nor tigns of strangulation Aial contrast-enhanced CT image
shows a ish Fiq. 83,3 ventral hernia containing small bowel loops and mesentery are present Discussion
Background viscus from its proper cavity into a saclike A hernia is commonly defined as the protrusion of
any formed by the same membrane with which the cavity is usually lined. Different types of structure
frequent in both anatomical distribution, 83.2 and indirect). Others include the most hernias have been
described according to their women being inguinal hernias (direct Fig Femoral hernia: occurs medial to
the femoral vein and posterior to the inguinal ligament: com- monly right-sided Ventral hernia: Midline
defects: umbilical, paraumbilical, men and epigastric, and hypogastric hernias. Umbilical is the (risk
factors: obesity, multiple pregnancies most common type, usually occurring in women ascites Lateral
defects- cle through the hiatus semilunaris Posterior defects-lumbar hernia: occurs either through
defects in the superior (Grynfeltt) (Fig. 83.4 or inferior (Petit) Jumbar intra-abdominal masses) -spigelian
hernia (Fig. 83.3): occurs along the lateral aspect of the rectus mus- triangle Fig. 83.4 (A.B)n The contents
of the ia of Grynfelit cn be conluned with a lipomatous tumor seen on CT(aroww) Note that this hernia
contains a portion of Gerota fasc

267 AVITY AD EROPERITONEUs vareCT Fig. 83.5 Asial CY ge of the pelvs shos a he ated bowel e in the left
obtat a (ane) Obturator hernia occurs through the obturator foramnen, usually in elderly multiparous
women (Fig 83.5) tecisional hernia: represents a delayed complication of abdoiminal surgery Abdominal
wall hermas are very common with aore than 500,000 operations performed in the United eai.
Unfortunately, surncal reair is associated with a high rate of complications (up to 201). collections, and
infection of pmsthetic materials The cinical diagnosis may be imaging plays a central with complicatioms
States every Mch as recurrence. fluid difficult in cases of obesity or abdominal scaning from a previous
surnery: hence i role in recognizing this pathologic entity and decreasing the morbidity associated
Clinical Findings and severity of abdominal hernias. Patient and discomfort with a palpable Clinical
presentation depends on the anatomical location) abnormal- presentation can range from vague
abdominal tenderness ity to a bowel obstruction Complications Major complications include
strangulation (Fig. 83.6) and incarceration (Figs. 83.1). both to bowel obstruction or perforation in case
of missed or delayed diagnosis. Incarceration refers to those hernias that cannot be manually reduced
because of progressive narrowing of the neck of the hernia sac. Strangulation refers to ischemia of the
protruded organ Other, less frequent nonsolid leading due to impaired blood solid (liver supply kidneys)
or complications include herniation of other organs, either (stomach, bladder) Fig. 83.6 Axial contrast-
enhanced CT image henia in the lelt lower quadrant descending colon. There is a small amount of fluid in
the her na ac. The bowel within the colon demonstrates mild wall thickening The colon These findings
are consistent with strangulation shows a ventral which contains a loop of distal to the henia is
underdistended.

268 TEACHING ATLAS OF ABDOMINAL IMAGING Most abdominal hernias are seen in adults and are
related to an acquired weakness of the abdominal wall. Abdominal trauma (iatrogenic or high-impact
blunt types of hernias by disrupting the abdominal wall musculature. Pediatric hernias and commonly
described in the groin as a consequence of a patent processus vaginalis. The forma ton of hernias in the
abdominal wall is promoted by any condition that increases pressure ing, and ascites muscular trauma)
may cause different are less frequent Etiology intra-abdominal obesity, coughing, chronic lung disease,
straining during defecation or voiding. heavy lin Currently, multidetector CT has become a maior
diagnostic tool for the evaluation and characteriza. tion of abdominal wall hernias, as it provides very
accurate anatomical details for surgical planning by means of its multiplanar after intravenous
administration of contrast material (to evaluate vascular supply). Either positive or negative oral contrast
material may be used to improve bowel loop visualization. Postural maneuvers sure (Valsalva) may be
useful in case of subtle hernias, CT is also very helpful in detecting early signs of complications, such as
incarceration, Imaging Findings performed reconstruction capabilities. Supine acquisition is usually
(prone or lateral decubitus) or maneuvers aimed to increase abdominal pres obstruction strangulation,
and bowel Treatment Surgical treatment is usually advised even in asymptomatic Different procedures
may be performed currently is the standard surgical technique for abdominal wall hernias. Recurrence
represents the most common complication after hernia repair, usually occurring 2 to 3 years procedure
Other complications may be the result of mesh infection or mesh shrinkage patients to avoid such as
laparoscopic suture and mesh repair, which complications after the surgical Prognosis The prognosis is
favorable in cases rence rate is very high (-50% of patients may require a second operation). prompt
diagnosis and surgical repair, although the recur- PEARL Valsalva maneuver, increasing abdominal
pressure, may be useful to detect subtle or unknown hernias at imaging PITFALL Imaging is unable to
always distinguish between an uncomplicated reducible henia and an incar- hernia, which may lead to
different management. cerated

*CASE 84 Clinical Presentation A 75-vear old man presents with severe. poorly localized abdominal pain
and diarhea Fig. 84.1 (A,B) Axial and (C) coronal CT images show bowel wall thickening of the descending
colon the splenic flexure with haustral pattern The affected segment of colon has a diffuse extending
from pericolonic fat stranding and loss of the normal targeted appearance with mild enhancement of the
typical mucosal layer Radiologic Findings Axial and coronal computed tomography (CT) images (Fig. 84.1)
show diffuse bowel of the descending colon the normal haustral pattern. mild enhancement of the
mucosal layer wall thickening flexure with pericolonic fat stranding and loss of has a typical targeted
appearance with extending from the splenic The affected segment of colon Diagnosis Acute mesenteric
ischemia 270

271 PERITONEAL CAVITY AND REROPENITONEUM Differential Diagnosis .Inflammatory bowel disease
(Crohn Infectious.colitis Prudomembranous colitis Neutropenic colitis disease, ulcerative colitis)
Ityphlitis) Discussion Background Mesenteric ischemia is a rare pathologic disorder (01s of emergency
room admissions) predomi nantly alfecting the older population of both genders (60 years), although it
may patients with predisposing risk factors. The prevalence of this condition is growing with the in
average life the superior mesenteric occur in younger increase expectancy celiac trunk flow can cause
Mesenteric in the transverse colon near Three arteries provide the blood supply to the bowel. the rtery
(SMA), and the inferior mesenteric atery (IMA) A decreane in blood ischemia and subsequent
reperfusion cell damage, leading to intestinal necrosis ischemia may be acute or chronic, arterial or
venous, diffuse or focal Iwatershed a areas vulnerable in the splenic Different etiologes, clinical
presentations, and imaging findings have flexure or in the sigmoid colon are extremely cases of
hypovolemia) identified for each of these pathologic entities been Clinical Findings Patients with acute
mesenteric ischemia usually present severe abdominal pan, diarrhea, nausea, and vomifing Patients
affected by chronic ischemia commonly experience postprandial abdamnal pain (intestinal angina)
weight loss, and food elevated serum lactate level wvoidance Laboratory tests may reveal leukocytns and
an (usually a late finding) Complications Ischemia causes tissue necrosis, which in turn may lead to bowel
perforation and subnequrnt peritunitis Etiology Major causes of acute mesenteric ischemia inchude
Arterial embolism (SMA most commonly (atrial fibrillation, valvular disease, and ventricular aneurysm
are the most important pre- affected), accounting for almost one third of the total cases disposing
Arterial thrombosts: responsible for-20 to 30% of cases, usually associated with preexisting ath-
erosclerasis of at least two intestinal Nonocclusive factors) vessels secondary to cardiogenic
vasoconstriction: prolonged sepsis, cocaine venous thrombosix (oral contraceptive mesenteric
hypoperfusion abuse, digitalis. and so on mostly described in patients shock or heart failure
hypertension Mesenteric and partal hypercoagulable state abdominal with portal use), intra-abdominal
sepsis, pancreatitis, and recent ralins

272 TEACHING ATLAS OF ARDOMINAL IMAGINC disease of two or mone ichemia mostly results from
severe atherosclerotic Dther causes are fibromuscular dysplasia, median arcuate compression of the
celiac trunk by the median arcuate ligament that syndrome Aortic dissection ligament syndrome
connects the left and and con- Complicated bowel obstruction or oventistention tie Chronic mesenteric
intestinal vessels extrinsic Takayasu arteritis) nght crura of the diaphragm) granulomatous vasculitis (in
particular nective tissue diseases such as Ehlers-Danlos assessment of mesenteric Imaging Findings
angiogiaphy demonstrates the presence of thrombi or emboli (filling defects) within he remains the gold
standard for the Currently, catheter-directed ischemia, as it directly vesselk However. CT angiography
(CTA) has become a key imaging technique for the evaluation of mesenteric ischemia, as it provides high
spatial resolution of the mesenteric vessels and additional (3D) datasets three-dimensional they allow
multiplanar reconstructions of the dis used information about hollow viscera. Moreover, the condition of
the surgical and interventional eased vessels -enhanced 3D magnetic resonance angiography (MRA) is
also widely insufficiency and planning because cau Contrast- especially in children. The recent
understanding of the relationship between renal nephrogenic systemic function Accurate CT imaging
impaired fibrosis has made MRA a less attractive modality in patients with renal during the arterial and
portal phases after the requires biphasic acquisition intravenous administration of contrast material to
demonstrate the patency of mesenteric arteries and veins. Unenhanced calcifications, intramural
hemorrhage, or hyperattenuating intravascular clotting. Positive or negative oral contrast material can be
administered to distend bowel loops. CT findings of mesenteric ischemia are varied images may add
valuable information. as they help to delineate vascular The most important include thickening: the most
observed in inflammatory bowel disease, edema Bowel common but least specific feature of bowel
ischemia (it may be infectious colitis, etc.) finding is not always predictive of poor outcome mostly
related to intramural hemorrhage, or superinfection (This because transmural ischemia usually results in
thinning of the bowel wall.) Dilatation: related to interniption of peristalsi5 in the injured bowel;
common findings in case of transmural extent of infarction Abnormal wall enhancement Mesenteric fat
stranding and fluid collections (the latter mostly seen in acute ischemia) Prieumatosis or
portomesenteric gas Treatment Acute mesenteric ischemia requires immediate surgical treatment
before irreversible damage occurs to the bowel Embolectomy with resection of the nonviable intestine
represents the emboli. Bypass grafting or Intra-arterial infusion of papaveritie (30-50 mg/h) is the
mainstay of treatment for nonocclusive mesenteric ischemia fintra arterial catheter-directed
thrombolysis Mesenteric venous thrombosis is treated with Patients standard surgical procedure for
endoarterectomy is usually performed in case of in situ thrombosis Chronic mesenteric ischemia may be
treated angioplasty, stent placement). anticoagulation. surgical intervention regardless of the with
endovascular techniques with signs of peritonitis need prompt etiology
273 VII CITRACT, PERITONEAL CAVITY, AND RETROPERITONEUM Prognosis The prognosis is poor with the
erosclerosis of mesenteric ve ortality rate ranging from 60 to 100%, Patients with severe ath r
angioplasty benefit from endovascular stent placement may to prevent acute ischemia. PEARL It is
imperative for the radiologist to include the mesenteric arteries and veins in their search pattern,
especially in the presence of colitis PITFALL eT findings in bowel ischemia may be extremely varied and
nonspecific. Suggested Readings Bradbury MS, Kavanagh PV, Bechtold RE, et al. Mesenteric venous
vasive imaging. Radiographics 2002:22:527-541 Chang RW, Chang JB, Longo WE Update in management
of mesenteric ischemia. World J Gastroenterol 2006;12(20):3243-3247 thrombosis: diagnosis and nonin-
Horton KM, Fishman EK. Muiti-detector row CT of mesenteric ischemia: can it be done? 2001:21:1463-
1473 Horton KM, Fishman EK. Volume-rendered 3D CT of the mesenteric anatomic variants, and
pathologic conditions. Radiographics 2002:22:161-172 Kirkpatrick ID, Kroeker MA. Greenberg HM.
Biphasic CT with mesenteric CT angiography in the evalu- ation of acute mesenteric ischemia: initial
experience. Radiology Klein HM, Lensing R. Klosterhalfen B. Tons C. Gunther RW. Diagnostic imaging of
mesenteric Radiology 1995:197:79-82 Landis MS Rajan DK, Simons ME, et al. Percuitaneous
management of chronic mesenteric ischemia outcomes after intervention. J Vasc Interv Radiol
2005;16:1319-1325 Rha SE Ha HK. Lee SH, et al. CT and MR imaging findings of bowel ischemia from
various causes. Radiographics 2000:20:29-42- MC, Angle JF, Leung DA, et al. CTA and MRA in mesenteric
ischemia: 2. Normal findings an Radiographics vasculature: normal anatomy 2003:229:91-98 infarc tion
primar Shih complications after surgical and endovascular treatment. AJR Am J Roentgenol 2007:188
462- 471 Shih MC, Hagspiel KD. CTA and MRA in mesen diagnis AIR Am J Roentgenol 2007:188:4eteric
ischemia: 1. Role in diagnosis and different -461 rasimhaiah 1.Diagnosis E1372 Deneuville M. Pradel JA,
Regent D. Bruel JM. Acute t nhanced CT. Radiology 1996:199:632-636 eaW.Khurana B, Jib H. Ros PR. CT
of acute bowel ischemia. Radiology 2003:226 635-650 and management of intestinal ischaemic
disorders. BMI 200 -1376 mesenteric ischemia: diagnosis w

*CASE 85 Clinical Presentation A 42 yearald man presents with abstominal cramp fever, and diarhes Fig.
85.1 (A) An axial contrastt enhanced CT imans bowel wall thickening and enhanoment o axial scan
showes pr shows severe the ascending colon (B) inferiori nent lymph nodes scattered in the mesenteri
fat, l reactive. This finding may favor the infectious etiology al colits. (C) Axial image through the pelvis
ment of the dstal large vasculature shows imohe bowel with prominent mesocal Radiologic Findings
obtained after the administration pericolonic fat stranding involving the majority of the ascending and
the entire transverse colon Scattered mesenteric lymph nodes are noted Axial computed tomography
(CT) images (Fig. 85.1) of the abdomen contrast show severe bowel wall thickening and of intravenous
Diagnosis Infectious colitis Differential Diagnosis Pseudomembranous colitis Crohn disease Ulcerative
colitis Ischemic colitis Radiation colitis Typhlitis 274

275 VCTRACY RIONAL CAITY AN) ETRORRTON Discussion Background infectious colitis is caused by a
variety of infectious agents and is described in bath immancom petent and immunocompromised or
hospitalized patients The lesians seen in thene patients may he either superficial isimilar to ulcerative
colitis) or transmural (as in Crohn disease diffise (eyto- megalovins and Escherichia coli) predominantly
right uded (as seen in colitis cauied by Yersinia Salmonella, amebiasis norchea binpsy are required for the
diagnosis, as there is a considerahle a colitides and tuberculasis or mosthly left sided (schistosomiais
shigellsis i syphilis, and lymphogranuluma venereum) Stool analysis, blood culture, and endoscopic
overlap in the CT appearance of herpes these clinical Findings Clinical presentation is extremely variable
acconding to the different etiologies Fatients may fe ihen asymptomatic or present with abduminal pain
and cramping nausea, fever, diarthea, and even dys- entery Complications perforation and subsequent
peritonitis Toxic mrgacolon repre described in pseudo Transmural involvement may cause sents an
uncommon complication of infectious coltis and is more commonly membranous colitis,
cytomegalovinus colitis, and amebiasis Etiology Different agents may cause infectious colitis, including
Bacteria Salmonella Yersinia, gonococcus, and Chlamydia trachomaris (rectal lymphogranuloma
venereumi Mycobacterium ingestion of contaminated milk poducts Virus: cytomegalovirus causing a
rane form of colitis typically Purasites: Entamocba histolytica is the most common cause of parasitic
colitis Other agents are Strongyloides stercoralis, Trichuri Pseudomembranous colitis is a particular form
of infection administration of broad-spectrum antibiotic ampicillin: chemotherapeutic agents microflora
favors the colonic overgrowth of toxin-producing Cloxtridium difficile typhi, E coli (enterohemorrhagic,
enteroinvasive). Shigella, Campylabocter tuberculosis may be involved in patients with pulmanary
tuberculosis or after seen in immunocompromised patients trichiuru, and Schistosoma occurring 1 to 6
weeks therapy (clindamycin, lincomycin, tetracycline, and may also be responsible). The resulting
alteration of normal after the Imaging Findings extremely nonspecific and mainly consist of the also
described in inflammatory bowel disease and ischemic colitis. Common features CT findings in infectious
colitis are typical inflam matory changes include Bowel wall thickening, a sensitive but average thickness
of the colonic the presence of nonsperific finding, in pseudomembranous colitis, the wall is 15 mm
between the thickened bowel folds) with preserved haustra (accordion sign due to contrast

Pericolons lat stranding Ruid collectionns (ascites s mare commonly observed in acute colitides
particularly in pseudo imembeanous colitis and cytomegalovirus colitis) 276 11ACHNG ATLAS OF
ABnoNAL MACING analysis must be performed to identify the infectious etiology toxic megacolon)
Surgery is Treatment Blood culture and stool required only in case of complications (perforation,
Prognosis The prognosis is usually favorable if patients are given an appropriate antimicrobial therapy
Careful history taking including inquiry regarding recent travel or antibiotic therapy, can be important in
the evaluation of patients with suspected infectious PLARI colitides PITFALL Imaging features viscera
demonstrate considerable overlap by imaging alone infectious colitides and other inflammatory
processes in the hollow which makes a precise etiology difficult to ascertain different . Suggested
Readings Ahualli J. The target sign: bowel wall. Radielogy 2005:234:549-550 Ash L Baker ME, O'Malley
CM Jr. Gordon SM, Delaney CP. Obuchowski NA. Colonic abnormalities on CT in adult hospitalized
patients with findings. A/R Am J Roentgenol 2006,186:1393-1400 Desai RK, Tagliabue JR, Wegryn SA,
Einstein DM. CT evaluation of wall thickening in the alimentary tract. Radiographics 1991:11:771-784
Horton KM Corl FM, Fishman EK. CT evaluation of the colon: inflammatory disease. Radiographics 2000
20:399-418 Miller FH, Ma I, Schalz Fl. Imaging features of enterohemorrhagic Eschenc coli colitis
Roentgenol 2001:177619-623 Murray JG. Evans S. Jeffrey PB, Halvorsen RA Jr. Cytomegalovirus colits in
CT features. AJR Am J Roentgenol 1995:165 67-71 Nakamura S lidaM. Tominaga M, Yao T. Hirata N.
Fujishima M. Salmonella s assessment with double-comtrast barnum enema examination in seven
patients. Radiology 199 Clostridium difficile colitis: prevalence and significance of Am LE Heiken jP
Westco Rady 19490 445-449 MA Gore RM. Colitis: use of CT findings in differential diagnos 84:537-540
RECello JP CT imaging of colitis Radiology Witber Harisinghani MG. Jhavent K. Varghese of the abomal
bowel 2006:240:623-638 wall Radiographics 2002:22:1093-110 Muelier PR. Alorthmic approach to CT
dia

*CASE 86 Clinical Presentation A 24 year-old man presents With ght lower quadrant pain, lever, and
nausea Fig. 86.1 (A,B) Axial contrast-enhanced CT images demon- strate appendiceal enlargement and
wall thickening (arrow) with stranding in the periappendicular fat (mesoappendix). (C) Coronal
reformatted image shows the thickened appendis along with periappendiceal inflammatory changes C
Radiologic Findings Axial and coronal contrast-enhanced computed tomography (CT) images (Fig. 86.1)
demonstrate appendiceal enlargement appendix). A calcified appendicolith is noticed within the
appendix. These findings are consistent with and wall thickening with mild stranding in the
periappendicular fat (meso acute appendicitis Diagnosis Acute appendicitis 277

278 TEACHNC ATLAS OF ARDOMINAL MALNG Inflammatory bowel disease (Crohn disease and ulcerative
colitis) ucocele (abnormal accumulation of mucus within the appendix) Adenocarcinoma
cystadenocarcinoma Carcinoid Differential Diagnosis Cecal diverticulitis Discussion Background disease
requires in young patients of both prompt surgical treatment to avoid complications: for this reason,
radiologists should be able to recognize this entity as promptly as possible Appendicitis represents the
most common cause of tight lower quadrant pain, usually occuring predominance. This higher male
genders with a slightly Clinical Findings anorexia, nausea lower quadrant pain and tenderness, fever,
Appendicitis usually presents with right and yomitine, However, the clinical presentation e variable
position of the appendix, the degree of inflammation, and the timing of onset can all con- tribute to a
difficult clinical diagnosis. Laboratory tests typically showa leukocytosis with can be protean and mimic
other pathologies. The left shift Complications Lumen distention may lead to perforation of the appendix
with consequent spillage of pus into the peritoneal cavity. Once perforation has occurred, appendicitis
may progress to abscess formation Less frequent complications are hepatic abscess and small bowel
obstruction Etiology Appendicitis is mostly the result of luminal obstruction by means of appendicoliths
material usually located at the appendiceal orifice) or lymphoid hyperplasia: other less common causes
are parasitic infestation (Ascaris and Strongyloides species), tumors, and foreign bodies. Appendiceal
obstruction results in the accumulation of fluid, luminal distention, venous engorgement, and bacte- rial
invasion leading to inflammation. Perforation may occur whenever the diagnosis is delayed (calcified
fecal Imaging Findings CT represents the most sensitive inmaging technique to evaluate appendicits and
its complications although ultrasound has been shown to be a reliable tool as well and should be
performed as initial imaging test in pediatric patients and should be considered in women of
childbearing age Magnetic resonance imaging may be used At CT. the normal appendix appears as a
small, tubular, commonly t-filled structure usually located at the ileocecal valve or posterior to the cecum
(a retrocecalppendix has been described in -65% of patients appendix as seen in acute appendicitis, is
dilated 6 mm in diameter) and distended by fluid or debris: its walls appear mildly early pregnancy its
length usually varies from a few centimeters to 20 cm. An abnormal enhancing and thickened. Other
imaging findings are periap

ON 279 W TECT PETONEAL CAVTY AND ETROP endicular inflammanary in the mesoappendis C nding
avymmetical cecal wall thickning and hmtaeay appendicoliths may be seen within the appendis in 25 to
40of cases The use of oral contrasu material at CT is well established, as it can helpfui to show the partial
lling of the diseased appendix head shaped area due to focal thickening of the cecal wall at the os of the
appendix. This finding When rectally administered, the contrast media depict an arrow called the
arrowhead sign, is specific for appendscito CT is also useful in assessing the presence of complications,
an abscess appears as a pericecal fluid collection surrounded by inflammatory periappendicular free air
or, less commonly, as pneumoperitoneum changes perforation is seen as small pockets of whereas
Treatment Surgical management is recommended as soon as passible to avoid complications, such as per
foration and abscess formation an abscess is present, management may include image-guided
percutaneous catheter draag and an interval appendectomy Prognosis The prognosis is usually favorable
if appendicitis is promptly recognized. Any delay in the diagno- sis may increase the mortality/morbidity
rate. Therefore, role in patient management cross-sectional imaging plays a central PEARL : The presence
of an appendicolith within the lumen of the appendix associated with periappen- dicular inflammatory
changes is highly suspicious for appendicitis PITFALL Periappendicular/pericecal inflammatory changes
(fat stranding. lymphadenopathy tions) without have been described in inflammatory bowel disease and
cecal diverticulitis fluid collec evidence of appendiceal abnormalities are not diagnnstic for appendicitis,
as they Suggested Readings Callahan MI, Rodriguez DP. Taylor GA. CT of appendicitis in children.
Radiology 2002:224:325- -332 Horton KM, Carl FM, Fishman EK CT evaluation of the colon 2000:20:399-
418 Kaiser S Frenckner B, Jorulf HK. Suspected appendicitis in children: US and CT-a prospective ran-
domized study. Radiology inflammatory disease Radiographics 2002:223:633-638 women: evaluation
with MR imatine, in quadrant pain and suspected appendicitis in premant experience Oto A. Ernst RD,
Shah R et al. Right-lower- Radiology 2005.234 445-451 RA Distal appendicitis: CT appearance and
diagnosis Radiolagy Rao PM, Rhea JT. Novelline 1997 204 709-712

*CASE 87 Clinical Presentation A 45 year old man with chronc abdominal and weight loss symptoms
presents with abdominal pain diarhea Fia. 87.1 An axial CT imaqe showa mild enhancement Fig. 87.2 An
asial CT image shows moderate wall thickening of the teminal of inflamed bowel loops have a straed
appne and peesent mild enhancement after admst tion of contrast medium. Thee is pro c of men- ter
vessels saunding the aed bow aega The wa and thickening of terminal ileal w) and cecum walls t the
ileocecal junction with mild peri-inflammatory fat stranding Radiologic Findings and coronal computed
tomography (CT) images (Figs obtained after oral administration of negative contrast material show
diffuse, imegular wall thicken ing of distal small bowel loops mostly affecting the terminal of the fat
surrounding the diseased bowel loops. The ileocecal junction appears affected as well. with mild Axial
87.1, 87.2, and 87.3) af the abdomen leum. There is moderate prominence cecal wall thickening
Diagnosis Crohn disease (regional enteritis/terminal ileitis/granulomatous ileoolitis Differential Diagnosis
Infectious enterocolitis, including tuberculosis Easinophilic enteritis Radiation enteritis Behcet disease
281

A 282 CHN ALAs or Fia. 87.3 Coranal CT mage f the abdm Age, t ndg vidy desenibed t the senl deam and
with ular wall thckingOther jejunel wel oos o the p mesgastrium.appear matory chnges chustered and
allecied Iy n Ulcerative colitis Diverticulitis appendicitis, which can be mimicked by acute iiestis
Carcinoima or lymphoma Acute Discussion Background Crohn disease is a chronic granulomatous
inflammatory condition of unknown etiology, occurmng in multiple discontiguous sites of the
gastrointestinal tract. Lesions may occur anywhere mouth to the anus but are mostly encountered in the
small bowel. especially the terminal colon (in 70%1 and, less commonly, duodenum (in 201 quently both
involved Incidence is higher are slightly more affected than men The most typical pathaingit feature is
the involvemen of all layers of the diseased howel In early stages, the submucosal layer edena: as the
disease progresses. the inflammation exiends transmurally, and mucosal lesions devrlop, At advanced
stages ulcers tend to become deeper. leading to simus tracts and fistu las Scaming and fibrosis are the
final results of this inflammatory process Despite the presence of these typical features, Crohn disease
cannor always be distinguished Irom ulcerative colitis -20 of patients present indeterminate findings
from the ileum i The small and large bowels ae fr and women 8011 among Caucasians in their 2nd to 3rd
decade affected by hwperplassa of lymphoid tissue and lymph aphthous Clinical Findings Clinical
presentation is exrmely variet extenst of the disease Mot patients eaperience oded vomitfv Codse ry
tests senim dial analys yplbmitemia hypakalemia tpocalcema Extraintinal station and scleing holng
depending on the acuity of onset abdominal pain and cal location an d diarrhea anoresia and weight loss
mone commoniy Bloody ay reveal anema leukcyis in activ n typical a and abmom include gallstones and
kainey stones, arth episer

v aTRACT PERITONEAL CAuITY AND RETOPERITONEL 283 Complications Major complications are
obstraction Ldue to stricturing abncess fo hstulas (entrrovesical, enteroenteric, enteromesenteric
enterocutaneous rectovaginal toxic megacolon mation, and perforation and perianal) Surgcal therapy
may be needed. atthough generally comervanive e attempted as Brst-line therapy. Diarrhea medical
therapy nutrition Patients with Crohn disease have an increased risk of developing coloreCtal cancer and
lymphoma compared with the general population prevents nutrient absorption and hence may cauner
mal- Etiology The etiology factors may all be involved. Poor remains uncertain. Several studies suggest
that and genetic autoimmune, infectious, have also been cited diet, tobacco use, and oral contraceptive
use Dther studies focus on the presence of mesemteric vascular alterations as potential causative factors
(damage to submucosal blood vessels), thus defining Crohn disease as a granulomatous vasculitis
Imaging Findings Conventional imaging modatities well as ultrasonography and (barium enema, small
bowel series, and enteroclysis studies as cross-sectional imaging techniques (CT and magnetic evaluation
of Crohn resonance IMR have been used for the disease. Both CT and MRI present some advantages
walls and in the sur- including the ability to demonstrate the extent of inflammation within bowel
rounding assessment of complications and fistulas media are advocated in the assessment of patients
with Crohn disease. either ingested or administered by means of a nasojejunal catheter for strictures and
fistulas fat; multiplanar capabilities are useful in the Intravenous (IV) contrast and intraluminal contrast
media (enteroclysis), The most important finding5 of Crohn disease are are helpful in the assessment of
the mucosa and the search Asymmetric/eccentric Stratified bowel wall thickening (4-5 mm) appearance
of the affected bowel loops (target or double-halo sign) with the inner and layers surrounding a
lypodense middle layer Prominent Luminal narrowing and prestenotic dilatation Fibrofatty outer
enhancement of the diseased walls after IV injection of a contrast medium proliferation (creeping fat)
surrounding the inflamed bowel loops recta due to hypervas- sign (dilatation, tortuosity, and cularity of
the affected mesentery) Abscess and phlegmons occurring in the mesentery, abdominal wall, psoas
muscles, and perianal Comb prominence of the mesenteric vasa region Fistulas and sinus tracts (better
demonstrated with positive oral Mesenteric lymphadenopathy contrast suspect carcinoma or
lymphoma) noninvasive diagnostic tool for the study of small bowel detection of early lesions in patients
with Crohn (3-8 mm; if > 10 mm, Capsule endoscopy disease and may be However, this represents a
promising very useful in the disease modality is contraindicated in the setting of strictures Treatment
Medical treatments aim to inhibit the inflammatory process, control infections, and avoid mal- goals
Corti Four different classes of drugs aminosalicylates, antibiotics. and are therefore administered to fulfill
these nutrition costeroids immunomodulators

*284 TEACHING ATLAS OF ABDOMINAL IMACING .Surgical treatment is required when medical therapy
has proved inadequate to control the disease The prognosis depends on the anatomical site and extent
of the disease. The recurrence high even in patients responding to medical therapy; moreover,-50% of
patieut require surgical treatment. Quality of life is affected by long-term medical treatment (e.g.,
corticsteroids causing osteoporosis rate is Prognosis osteomyelitis, or increasing susceptibility to
infections). Crobn disease has a bimodal distribution in terms of age at presentation, with the majority of
patients presenting in the 2nd to 3rd decade and a small cohort presenting in the 7th to 8th decade
PEARL PITFALL enhancement or obscure small early demonstrate fistulas may hide mural Positive oral
contrast lesions Negative oral or extramural fluid collection. such as an abscess (high attenuation)
contrast (low attenuation) may not be able to easily Suggested Readings Desai RK, Tagliabue JR. Wegryn
SA, Einstein DM. CT evaluation of wall thickening in the alimentary tract. Radiographics 1991:11:771-784
Furukawa A, Saotome Yamasaki M, et al. Cross-sectional imaging in Crohn disease. Radiographics
2004:24:689-702 Hara AK, Leighton JA, Sharma VK, Heigh RI. Fleischer DE. Imaging of small bowel son of
capsule endoscopy,standard endoscopy, barium examination, and CT. Radiographics 2005:25 697-711
disease: compari- Horton KM, Corl FM, Fishman EK. CT evaluation of the colon: inflammatory
2000:20:399-418 Koh DM, Miao Y, Ching R. et al. MR imaging evaluation of the activity of Crohn's
disease. AjR Am Roentgenal 2001:17 1325-1332 Madureira Al The camb sign. Radiology 2004:230:783-
784 Paulsen SR Huppich JE. Fletcher JG. et al. CT enterography as a diagnostic tool in evaluating
disorders: revigw of clinical experience with over 700 cases. Radiographics 2006:26.641-657 Prassopoplc
PPapanikolaou N. Grammatikakis J. Rousomoustakaki M. Maris T. Gourtsoyannis N MR enterysis imaging
of Crohn disease. Radiographics 2001:21:5161-5172 Sailer Pelschek P.Schober E, et al Diagnostic value of
CT enteroclysis compared with conventiona enterhyss in patients with Crohn's disease. AJR Am ]
Roentgenol 2005,185:1575-1581 wi isobis iF Denstman F). Crohn disease state of the art R ogy 1997 202
597-610 Wold PFletchor 1G Johnsan CDSandburm W Assessment of small bowel Crohin disease: non
asive peroral C enterography.compared with other imaging udy Radiolgy 2001 229 275-281 disease.
Radiographics small bowe methods and endoscopy-leasibi
286 HACHuC ATLAS OF AenOMINAL BAGING Differential Diagnosis Polypoid neoplasms (benign or
malignant Inverted Meckel diverticulun Intussusception is the invagination of a bowel loop with its
mesentery (intussusceptum) into the lumen of a contiguous loop (intussuscipiens) This abnormality
mostly occurs in infants (40% at an age of 6-7 months) but also in adults (5%), accounting for 13 of all
bowel obstructions Intussusceptions may affee aimost any part of the small and large bowel, so that
different tyvpes have been described: ileocolic, ileo- ileocalic (together they account for 90% of cases in
childhood), ilepileal, and colocolic. Intussusceptions Discussion Background may be further classified as
idiopathic or nonidiopathic if a causative lead point is present Pediatric patients usually present with
colicky abdominal pain, vomiting. and the typical red "cur rant jelly" stools (ie. stools mixed with blood
and mucus), although most positive for occult blood. Physical examination may reveal a mass upon
palpation of the abdomen Fever is rarely a presenting symptom and may be the result of a complication,
such as sepsis. Adults present with nonspecific complaints similar to symptoms of bowel obstruction.
Clinical Findings infants have only stools Complications f not promptly resolved. intussusception may
result in obstruction or perforation of the affected loops as soon engorgement, ischemia, and necrosis
Intussusceptions may also recur (10% of cases), especially within 48 hours the compression of the
mesenteric fold of the intussusceptum leads to venous after radiologic reduction Etiology Almost 90% of
all pediatric intussusceptions present no lead point: these idiopathic related to hypertrophy of lymphoid
tissues secondary to viral infection. Other predisposing are cystic fibrosis and recent upper cal lead point
can be identified: Meckel diverticulum, intestinal polyps (Peutz-Jeghers cases may be factors respiratory
infections. In the remaining 10% of cases, a mechani- juvenile polyposis, etc.) blunt abdominal trauma,
intestinal lymphosarcoma, foreign bodies, heman- syndrome -Schonlein purpura (xmall bowel
hematomas) represent the most common giomas, cysts, and Henoch causative agents, In adults (eg,
lipoma, adenomatous sion) acting as a lead point. An increasing number of idiopathic lead point) have
been identified at cross-sectional imaging, CT, and in the absence of symptoms or mechanical
obstruction and are felt to be physiologic almost all symptomatic intussusceptions are due to an
underlying disease polyps, metastases. lymphoma, adenocarcinoma, and postoperative adhe-
intussusceptions (no evidence of a Imaging Findings Abdominal plain films (both with aa al intestinal gas
pattern, or may have features suggestive supine and prone views) may be normal, m a soft tissue mass
wel obstruction.

VIl GITRACT, PERITONEAL CAVITY, AND RETROPERITONEUM 287 The typical ultrasound finding of
intussusceptions is the target or doughnut sign on transverse sections due to the presence of a central
hyperechoic area surrounded by a hypoechoic edema tous bowel loop: on longitudinal sections, this
abnormality displays a pseudokidney or reniform appearance. The same findings may be described at CT,
but this imaging technique better depicts the surrounding inflammatory changes, the complications
related to this abnormality, as well as the underlying causes, Small bowel intussusceptions without a
lead point are more common than intussusceptions with a lead point and usually appear shorter and
smaller in diameter, Most large bowel intussus- ceptions (50%) have a lead point, such as
adenocarcinoma, lymphoma, metastases, lipoma, and adenomatous polyps. CT can identify the
underlying lesions but may not be able to characterize them. Treatment In children, surgery must be
performed in case of perfóration, peritonitis, or hypovolemic shock. IE patients are clinically stable,
reduction by means of barium or air with fluoroscopic guidance may be successfully performed.
Ultrasound has been increasingly acknowledged as an afternative method for the reduction of childhood
intussusception because of its high success rate (80% ) and its avoidance of radiation exposure. In adults,
surgical removal of the underlying cause is required to solve intussusceptions Prognosis The outcome is
favorable if the intussusception is promptly recognized and properly treated. The recurrence rate after
barium enema reduction or ultrasound-guided reduction is 6 to 10 % , with half of the cases.occurring
within 48 hours. PEARL The presence of a bowel-within-bowel configuration with or without the typical
inflammatory changes due to the compression of the mesenteric fold and vessels is pathognomonic of
intussusception. PITFALL Increasingly seen physiologic intussusceptions on cross-sectional imaging can
lead to unnecessary imaging and patient concern Suggested Readings Bisset GS lI, Kirks DR.
Intussusception in infants and children: diagnosis and therapy. Radiology 1988: 168(1):141-145 Boudiaf
M, Soyer P, Terem C, Pelage JP Maissiat E, Rymer R. er evaluation of small bowel obstruction.
Radiographics 2001:21:613-624 Bramis I, Griniatsos J, Papaconstantinou 1, et al. Emergency helical CT
scan in acute abdomen: a case of intestinal intussusception. Ulus Travma Acil Cerrahi Derg.
2006;12(2):155-158 Cois A. Pisamu A Pilloni L, Uccheddu A. Intussusception of the appendix by
mucinous eystadenoma: report of a case with an unusual clinical presentation. Chir Ital 2006:58(1):101-
104

*CASE 89 Clinical Presentation A 72-year-old man presents with diffuse abdominal pain. Fig, 89.1 (A)
Axial CT Image shows gas in the portal vein branches (peripherally located, as opposed to.central gas in
the pneumobilia) and free gas under the anterior abdominal wall (8)) Lower axial CT image in the same
patient shows curvilinear gas collections in the wall of the distended bowel loops. Radiologic Findings
JAISSeu bowel ischemia show the presence computed tomography (CT) images in a patient with of
hypodense curvilinear gas lucencies in the bowel wall branches (Fig. 89.1). Diagnosis siso1eunaud jamog
Differential Diagnosis 5jeunsau spi035 Bowel perforation estinal obstruction bowel angesadois 682

290 TEACHING ATLAS OF ABDOMINAL IMAGING Discussion Ihe presence of gas within the bowel wall is
termed pneumatosts intestinalis. Recognition of pneu matosis intestinalis in symptomatic patients is
important because it may be the only finding of an underlying pathology, which may not be always
obvious on imaging Background Pneumatosis intestinalis can be primary (idiopathic) or secondary (due
to underlying causes) Clinical Findings Asymptomatic Sudden onset of abdominal pain Bloody diarrhea
Abdominal distention Vomiting pain Obstipation . Back Complications Hemodynamic shock Functional
intestinal obstruction Peritonitis Preumoperitoneum Etiology Mucosal disruption from any cause Chronic
obstructive pulmonary disease (causes primary form) Bowel ischemia Bowel perforation Postoperative
Necrotizing enterocolitis Caustic and corrosive ingestion bowel Intestinal obstruction Inflammatory
bowel disease Celiac sprue Whipple disease Connective tissue disorders Inflammatory or infectious
Chemotherapy, cancer, and bone marrow transplantation Immunosuppression due to any cause
Idiopathic enterocolitis Imaging Findings Plain radiographs demonstrates pneumatosis in all cases with or
without an underlying cause (Figs. 89.2, 89.3) can reveal the presence of pneumatosis and
portomesenteric gas. CT clearly
VII GI TRACT, PERITONEAL CAVITY, AND RETROPERITONEUM 291 Fia. 89.2 Axial noncontrast CT image in a
45-year-old man with a subacute ence of preumatosis Fig. 89.3 Aial 32-year-old-man with vague
abdominal pain shows the presence of prieumatosis and retro- peritoneal gas in the right
hermiabdomen, nonconthast CT image in a small intestinal obstruction shows the pres- The presence of
liner or cystic gas collections in the is the hallmark of this condition wall nondependent portion of the
bowel Multiple vesicles < 1 cm are more commonly seen in primary within the bowel wall are
characteristic of the second- pneumatosis, whereas liner gas collections ary and more severe form. The
presence of free fluid, portomesenteric gas, bowel wall thickening and fat stranding should patients and
prompt an extensive search for the underlying cause in otherwise symptomatic necessitates contrast CT
examination to rule out bowel ischemia. Patients with bowel ischemia also have multiple dilated bowel
wall, which appears thickened with lesser mucosal enhancement The presence of a transition point
between dilated and tinal obstruction intestinal loops because of associated paralytic ileus and edema of
the nondilated bowel loops suggests an intes- Treatment The primary form of bowel pneumatosis does
not need any treatment unless it is associated with underlying abnormality resolves pneumatosis in
patients complications. The treatment of any with the secondary form. Prognosis The prognosis is
excellent for primary pneumatosis cystoids intestinalis. Secondary pheumatosis with the presence of
portomesenteric gas is associated with a grave prognosis and higher mortal ity rates PEARL The presence
of pneumatosis and associated diagnosis findings in symptomatic patients can suggest the

292 TEACHING ATLAS OF AlBDOMINAL IMAGING PITFALL The underlying condition leading to
pneumatosis intestinalis may not always be apparent on imaging Suggested Readings Kernagis LY. Levine
MS, Jacobs JE. Pneumatosis intestinalis in patients CT findings with viability of the bowel. AJR Am J
Roentgenol with ischemia 2003:180(3):733-736 correlation of St Peter SD, Abbas MA, Kelly KA. The
spectrum of pneumatosis intestinalis. Arch Surg 2003:138(1): 68-75 Wiesner W, Khurana B, li H. Ros PR.
CT of acute bowel ischemia. Radiology 2003:226(3):635-650

*CASE 90 Clinical Presentation A 72 year-old woman presents with abdominal distention, fullness, and
pain in the right iliac fossa Fig. 90.1 (A) Axial contrast enhanced CT image shows the presence of a cystic
lesion with thick walls in the right illac fossa with surrounding fat stranding and free fluid. (8) Axial
contrast enhanced CT image of the liver in the same patient thows scalloping of the liver margins and
ascites with peritoneal Ithickening Radiologic Findings Contrast computed quadrant with ascites and
scalloping of the liver margins (Fig. 90.1) tomography (CT) images show the presence of a cystic lesion in
the right lower Diagnosis Malignant mucocele of the appendix with pseudomyxoma peritonei
Differential Diagnosis Abscess Normal fluid-filled ileal loop Tuberculous ascites Peritoneal metastases
Malignant ascites cystic lesions Ovarian Bowel perforation Peritoneal mesothelioma Meckel diverticulitis
with perforation 293

294 TEACHING ATLAS OF ABDOMINAL IMAGING Discussion Mucocele of the appendix is the
accumulation of mucus with luminal distention of the appendix The reported Background variety results
from occlusion of the appendiceal luminal opening into the cecum from a phlebo- ith or impacted feces.
A malignant mucocele results from mucin-secreting adenocarcinoma of the appendix. Either a benign or
a malignant mucocele can result in the development of pseudomyxoma peritonei on rupture incidence is
0.2 to 0.5% of appendectomies, It can be benign or malignant. The benign Clinical Findings Chronic pain
in the abdomen Abdominal distention Mass in the right iliac fossa Intermittent colicky pain due to
intussusception Vomiting Complications Rupture of the mucocele Development of pseudomyxoma
peritonei Intestinal obstruction Intussusception Etiology Obliteration of appendiceal lumen from any
cause Imaging Findings On ultrasound, the mucocele may appear as a thin-walled cystic mass in the right
iliac fossa, attached to the cecum, filled with out septa and polypoidal luminal projections, and with
posterior acoustic enhancement. The pres- ence of a solid mass adenocarcinoma. Similar findings are
seen with CT, with the mucocele appearing as a low-attenu- ating structure in the right lower abdomen.
Calcifications within an unruptured mucocele can be curvilinear or plaquelike and can be appreci- ated
on plain radiographs of the abdomen measuring up to 15 cm reported in the literature. A ruptured with
internal echoes and fixed bowel loops. In patients peritonei, diffuse coarse internal echoes within the
ascitic fluid can be seen with scattered calci fications and scalloping on the peritoneal surfaces of the
abdominal organs fucent globules of uniform size that calcify in the course of time and shift with
movement are pathognomonic of the myxoglobulosis variant of a mucocele. .The presence of mucinous
nodules within the remnant umbilical vein is a feature that diffuse internal echoes in the dependent
portion with or with- within the nondependent portions of the lesion indicates the possibility of (Fig.
90.2) The size of the lesion varies, with lesions mucocele shows fluid-density ascites with the
development of pseudomyxoma Multiple solid trans- differentiate pseudomyxoma peritonei from
ascites. helps to

VI GITRACT, PERITONEAL CAVITY, AND RETROPERITONEUM 295 Fio. 90.2 Asial contrast-enhanced CT
image in a different patient reveals a cystic lac fossa. The lesion shows the presence of calcifications in its
wall lesion with thick walls in the right Treatment Surgical resection of the appendix is the treatment of
choice for an unruptured mucocele ol the appendix. In patients with a ruptured mucocele with the
development of pseudomyxoma perito nei, wide peritoneal stripping is done for cytoreductive
intraperitoneal Systemic chemotherapeutic agents can be used in resistant discase and recurrent disease
followed by heated intraoperative surgery chemotherapy with mitomycin C, doxorubicin, and fluorouracil
(5-FU) Prognosis Patients with an unruptured pseudomyxoma resistant disease, higher mortality, and
posttreatment mucocele of the appendix have an excellent prognosis. Patients with peritonei and
mucinous adenocarcinoma have a worse prognosis with treatment- complications PEARLS A cystic
oblong Scalloping of the peritoneal mass in the right iliac fossa should include mucocele in the
differential surfaces of abdominal organs represents pseudomyxoma peritonei PITFALL between benign
Imaging cannot always differentiate appendix and pseudomyxoma peritonei. and malignant causes of
mucocele of the Suggested Readings Dachman AH, Lichtenstein JE, Friedman AC. Mucocele of the
appendix and pseudomyxoma AIR Am J Roentgenol 1985:144:923-929 Madwed D, Mindelzun R. Jeffrey
RB Jr. Mucocele of the appendix: imaging findings Roentgenol 1992:159:69-72 Sasaki K, Ishida H,
Komatsuda T. Appendiceal mucocele: sonographic findings. Abdom Imaging 2003:28:15-18 peritonei, AIR
Am J
* CASE 91 cand discomfort associated with a ti Clinical Presentation lower abdominal pain A 74-year-old
man presents with change bowel habits Fig. 91.1 An aial contrast-enhanced CT image shows dif fuse
diverticuloshs, most significant in the sigmoid colon, with evidence of related mild muscular
hypertrophy. No pericolonic inflammatory changes are noticed. Radiologic Findings tomograplıy (CT)
image (Fig. 91.1) shows diffuse diverticulo- evidence of related mild muscular hypertrophy. No An axial
contrast-enhanced computed sis, most significant in the sigmoid colon, with pericolonic inflammatory
changes are noticed Diagnosis Diverticulosis Differential Diagnosis Inflammatory bowel Colonic
carcinoma disease (Crohn disease and ulcerative colitis) Small bowel diverticulosis Discussion
Background Diverticula are outpouchings of the intestinal walls, involving either all the layers (true
divertic- ula) or just the mucosal and submucosal layers (false diverticula). Diverticular disease has been
described anywhere in the gastrointestinal (GI) tract but is most frequently (95-98%) due to the higher
segmental through weaker points of the colonic walls, which and other nutrient vessels colon present in
the sigmoid intraluminal pressures. These diverticula usually form represent the natural openings for the
vasa recta 296

29 V GI TRACT, PERITONEAL CAVITry, AND RETROPERITONEUM Fig. 91.2 (A8) Axial contrast-enhanced CT
imanes show diffuse bowel wall thickening of the sigmold colen with su undng inflammatory mesenteric
fat stranding. These findings ane consistent with sevenre diverticulltis Clinical Findings Diverticulosis is a
common, usually asymptomatic condition. Patients with multiple e present with different symptoms
according to their location: clinical presentation includes left diverticula may most frequently diarrhea,
consti- lower abdominal pain, nausea, and changes in bowel habits, including pation, and tenesmus
Complications Major complications of the neck of a diverticulum by stool, inflammation, or ingested
material, causing subsequent micro perforation of a diverticulum and associated pericolic inflammation
(Fig. 91.2). Inflammation may resolve or progress, causing Or vagina), abdominal obstruction, or free
perforation. include GI bleeding fmelena, hematochezia) and diverticulitis due to occlusion abscess
formation, fistulas (to local structures, eg.. the bladder, uterus Etiology Diverticulosis is related to the
low-fiber/high-fat diet typical of industrialized countries leading to abnormal peristalsis, intestinal
dyskinesia, and increased intraluminal pressures. Its incidence increases with age; in fact, colonic
diverticular disease is uncommon in people younger than 40 years. Genetics may play a predisposing
role, as left-sided populations diverticula are usually described in Caucasian seen in persons of Asian
whereas right-sided diverticula are typically descent Imaging Findings Water-soluble contrast enemas
represent a safe but somewhat outmoded imaging study for diver- ticulosis On CT, diverticula appear as
multiple, air-filled outpouchings of the intestinal walls ranging in size from 2 to 3 mm to 2 cm (giant
diverticula have been in the sigmoid colon; the affected intestinal segment may have thickened walls due
to muscular hypertrophy diverticulitis (wall thickening and pericolic fat stranding), abscesses (fluid
collections with periph- eral enhancing rims), and perforation (extraluminal free air). Colovesicular
fistulas are suggested by the presence of gas in the bladder or focal bladder wall thickening in close
proximity to the reported but are rare) and usually located CT is also useful to evaluate the complications
of diverticular disease, such as acute affected bowel with loss of dividing fat planes segment of the
298 TEACHENG ATLAS OF ARDOMINAL IMAGING Treatment Surgery is suggested after Conservative
treatment is advised in cases of asymptomatic diverticula Tepeated bouts of acute diverticulitis and may
be required in case of fistulas, obstruction, and sepsis Prognosis The prognosis is usually complications
favorable, barring severe PEARL Diverticulitis may be present even though diverticula are not
radiographically evident. PETFALL findings; therefore Diverticulitis and a perforated colon cancer may
have overlapping gastroenterology consultation is usually performed by the referring exclude the
possibility of malignancy imaging physician for endoscopy to Suggested Readings Chintapalli KN, Esola
CC, Chopra S, Ghiatas AA, Dodd GD IIL Pericolic mesenteric lymph nodes: an aid in distinguishing
diverticulitis from cancer of the colon. AJR Am J Roentgenol 1997:169:1253-1255 Horton KM. Cori FM,
Fishman EK. CT evaluation of the colon: inflammatory 2000:20:399-418 disease Radiographics Jarrett TW,
Vaughan ED Jr. Accuracy of computerized tomography in the diagnosis of colovesical fistula secondary to
diverticular disease. J Urol 1995:153:44-46 Padidar AM. Jeffrey RB Jr, Mindelzun RE, Dolph JF.
Differentiating sigmoid diverticulitis from on CT scans: mesenteric inflammation Silva AC. Wellnitz CV,
Hara AK. Three-dimensional virtual dissection at CT colonography: unraveling the colon to search for
lesions, Radiographics 2006:26:1669-1686 carcinoma suggest diverticulitis. AJR Am J Roentgenol
1994;163:81-83 Thomas S, Peel RL, Evans LE, Haarer KA Radiographics 2006:26:1869-1872 Zaidi E. Daly
B, CT and clinical features of acute diverticulitis in an urban U.S. population: rising frequency in young,
obese adults, AJR Am J Roentgenol 2006;187:689-694 Best cases from the AFIP: giant colonic
diverticulum.

* CASE 92 Clinical Presentation A 70-year-old patient presents with constipation and hematochezia Fig.
92.1 Axial contrast-enhanced CT image of the sigmoid colon shows an enhancing "apple core" lesion
causing short segment narrowing (arrow) Fig. 92.2 Axial CT image of the upper abdomen in the same
patient shovws a peripherally enhancing hypodense hepatic metastasis in the liver perhepatic (arow).
Also seen are malignant ascites Radiologic Findings Axial contrast-enhanced computed tomography (CT)
images through the pelvis "apple core" lesion causing short 92.2 shows a hypodense malignant
perihepatic ascites. show an enhancing Figure segment narrowing in the sigmoid colon (Figs. 92.1, 92.2),
patient, peripherally enhancing hepatic metastasis in the same along with Diagnosis Adenocarcinoma
sigmoid colo hep metastasis Differential Diagnosis Colitis Diverticulitis Inflammatory fibrous polyp Benign
stricture due to inflammatory bowel disease or ischemiah or smooth muscle hypertroplhy from chronic
diverticulosis 299

300 TEACHING ATLAS OF ADDOMINAL IMAGING Discussion deaths in the United States the Background
are the second most common cause of cancer-related Colorectal cancers and Europe. The sigmoid colon
is the second moSt common site of colonic malignancy following rectum habits, weight loss Findings on
clinical -positive stool, iron levels Clinical Findings Symptoms include abdominal pain and distention,
constipation, change in bowel and symptoms related to disseminated metastatic disease and laboratory
examination include a palpable abdominal or rectal mass, heme- deficiency anemia. melena.
hematochezia, abnormal liver function tests and raised carcinoembryonic antigen (CEA) Complications
metastatic disease, fistula formation, sec- Complications ondary infection leading to abscess formation,
local invasion include intestinal obstruction, perforation, and intussusceptions. Etiology advanced age (>
50 years), personal and/or family his- Risk factors for colorectal carcinoma tory of colorectal
adenomatous polyps or cancers or polyposis syndromes, hereditary nonpolyposis colorectal cancer
(HNPCC, or Lynch syndrome), inflammatory bowel disease, low-fiber/high-fat diet from animal sources,
physical inactivity, smoking include and heavy use of alcohol. Imaging Findings Annular and semiannular
lesions tend to be more common than They also tend to have extraserosal extension and lymph than
polypoidal the appearance of an apple core lesion is a typical appearance of sigmoid colonic carcinoma.
Radiologic criteria for polypoidal lesion being malignant to 2 cm in diameter, retraction at the base of the
polyp, and rate examinations. Pericolonic and mesenteric fat stranding and to the lesion suggest
extraserosal spread of malignancy or surrounding inflammatory changes. The mesentery should be
assessed for lymphadenopathy. Carpet lesions in colon cancer present as asymmetric The liver is the
most common organ involved in metastatic disease. Other are the lungs, bones, and brain. Hepatic
metastases are most commonly hypodense, best visualized in the portal venous phase. Metastases from
mucinous colon cancers can be cystic and Widespread peritoneal carcinomatosis is more frequently seen
with mucinous cancers polypoidal and carpet (plaquelike) node metastases more commonly thickening
with luminal narrowing giving lesions lesions. Focal and irregular bowel wall are irregular outline, broad
base, size > 1 significant growth between two sepa- higher attenuation of fat adjacent nodular wal
thickening usual sites of metastases calcified. Treatment .Treatment depends on the stage of malignancy.
In early stages without serosal involvement mesenteric lymphadenopathy , the typical treatment is
surgical resection. Chemotherapy is con- and aidered for patients with more advanced disease. Adjuvant
radiation therapy is useful for the

301 VIL GI TRACT PERITONEAL CAVITY, AND RETROPERITONEUM treatment of tumor with spread to
adiacent organs. Surgical resection of hepatic metastases is performed in some patients. Prognosis The
survival rate depends on the initial presentation of the tumor. More than 90% of patients with early
stage disease have a 5-year survival rate, whereas <5% of those with metastatic disease do PEARL A
discrete rowing in the intraluminal soft tissue mass and irregular bowel wall thickening with luminal nar-
setting of mesenteric lymphadenopathy favor the diagnosis of carcinoma. PITFALL Colonic mucosal
lesions are not well seen on cross-sectional imaging, and preparation or distention for CT colonography
can make There is also imaging overlap between colon cancer bowel inadequate early colon cancers
difficult to identify and other conditions, such as diverticulosis and diverticulitis. Suggested Readings
Horton KM, Abrams RA, Fishman EK. Spiral CT of colon cancer: imaging features and role in manage-
ment. Radiographics 2000:20:419-430 Jang HJ. Lim HK, Park CK, Kim SH, Park IM, Choi YL Segmental wall
thickening in the colonic loop distal to colonic carcinoma at CT: importance and histopathologic
correlation. Radiology 2000:216(3): 712-717 Skucas J, Spataro RF, Cannucciari DP. The radiographic
features of small colon cancers. 1982;143:335-340 Radiology

* CASE 93 Clinical Presentation A 69-year-old man presents with abdominal pain and gastrointestinal (GI)
bleeding Vitrese WIL 1500 200 Fig. 93.1 (A-C) CT colonoscopy with insufflated carbon dioxide in the
colon. Axial CT images of the pelvis show a small isodense lesion protruding in the wall of the sig- moid
colon, consistent with a sessile polyp (arrow). Radiologic Findings Axial images from computed
tomography (CT) colonography show a small isodense protruding arising from the wall of the sigmoid
colon (Fig. 93.1) lesion Diagnosis Sigmoid colonic polyp 302

303 Vit GI TRACT, PERITONEAL CAVITY AND RETROPERITONEUM Differential Diagnosis Colonic
adenocarcinoma Inverted diverticulum Carcinoid Metastases Lipoma . Leiomyoma Foreign bodies
Discussion Background Polyps are protruding intraluminal lesions arising from the epithelial lining of the
GI tract, mostly affecting the sigmoid colon and rectum. These lesions more commonly occur in elderly
men, but they have also been noticed in younger patients affected by familial inherited polyposis
syndromes such as Gardner/familial adenomatous Peutz-Jeghers and Cowden syndromes
(hamartomatous polyps), and juvenile polyposis (inflammatory polyps) At hystologic analysis, polyps are
divided into inflammatory, hamartomatous, and adenomatous, the latter features for and Turcot
syndrome (adenomatous polyps), syndrome polyposis syndrome being the only premalignant type.
Morphology and size are the most reliable predicting malignant degeneration. Clinical Findings Clinical
presentation mainly relies on the size and location of the polyps; patients can be asymptom atic or
present with mia (hypersecretory changes in bowel habits, GI bleeding or mucous rectal discharge, and
hypokale- syndrome usually described in villose adenomatous polyps). Complications detection of
adenomatous polyps is mandatory, as these lesions are death in the United States Prompt known to be a
precursor of colonic cancer, one of the leading causes of cancer Etiology Hyperplastic inflammatory
Adenomatous polyps are premalignant: the sequence of morphologic, molecular, and genetic steps that
lead to malignant transformation is knoWn as the adenoma-to-carcinoma sequence polyps are focal
nonneoplastic proliferations of colonic epithelium. Imaging Findings from the colonic walls; however
polyps 1 cm and has been overwhelmed by resonance (MR) colonography. CT colonography can depict
the colonic wall in a manner similar to that seen with aptical endos copy: current software allows for the
creation of two- and three-dimensional reformatted images, and the specificity of this technique.
Research is ongoing to establish At barium enema, polyps are described as filling defects arising
technique has revealed low sensitivity to detect introduction of colonoscopy as well as of CT and
magnetic this the improving both the sensitivity

the best way to prepare patients for CT colonography: some experts advocate a traditional b preparation
akin to what is done for optical colonoscopy, with contrast and postprocessing subtrac tion techniques
used to assess the mucosa 304 TEACHING ATLAS OF ABDOMINAL IMAGING bowel The overall experience
with MR colonoscopy is still limited. Although these imaging studies have proved to be very promising,
optical colonoscopy remains the most.reliable technique for the detection of polyps Treatment
Endoscopic malignant transformation of adenomatous polyps removal is advised to prevent Prognosis
The prognosis is favorable if these premalignant lesions are promptly recognized and removed. PEAR
Polyps 2 1 cm in size should be referred for biopsy PITFALL distention may be potential problems in the
The presence of fecal masses and insufficient colonic identification of colonic polyps. Suggested Geenen
RWF, Hussain SM, Cademartiri F, Poley J-W, Siersema PD, Krestin GP. CT and MR colonog- raphy: scanning
techniques, postprocessing, and emphasis on polyp detection. Radiographics 2004 Readings 24:18e Hara
AK. Johnson CD, Reed JE, et al. Detection of colorectal assessment of sensitivity and specificity. Radiology
1997:20 lyps with CT colonography: initial 59-65 Macari M, Bini E. Jacobs SL, Lange N, Lui YW. Filling
defects at CT tive lesions 2003:23:1073-1091 Macari M, Bini E), Jacobs SL, et al. Colorectal polyps and
cancers in asymptomatic average-risk patients: evaluation with CT colonography. Radiology onography:
pseudo- and diminu- Radiographics good), polyps (the bad), flat lesions, masses, and carcinomas (the
ugly). 2004:230:629-636 Thompson WM, Foster WL, Paulson barium enema examination, Radiology
2006:239:139-148 EK, et al. Rocky causes of errors in polyp detection at air-contrast

* CASE 94 clinical Presentation A 45-year-old man presents with abdominal cramps and rectal bleeding
Fig. 94.1 (A-C) Axial CT images of the abdomen show mild diffuse wall thickening of the ascending
surrounding pes-infiammatory charnges: Multiple smal ymph nodes are noticed, The thickened wall
presents a typical stratified appearance withh negative attenuation (fatty halo sign) of the middle layer
and mild hyperdern- sty of the inner and outer layer. No fluid collections are He frontinued) calon with
Radiologic Findings Axial and coronal computed tomography (CT) images of the abdomen obtained after
administration bowel. of positive oral contrast medium (Fig. 94.1) show diffuse wall thickening of the
entire large with multiple small lymphadenopathies mostly located The mesentery in the right quadrant
close to the ascending presents mild fat stranding colon. There are no fluid collections Diagnosis
Ulcerative colitis 305

306 TEACHING ATLAS OF ABDOMINAL IMAGING Fig. 94.1 fcontinurd) (D,E) Coronal CT images of the
abdomen show the typical foreshortened, narrowed, and ahaustral appearance af th affected colon in
ulcerative coltis All the segments of the large (backwash feits). invohed. Note the involvement of the
terminal in bowel are simlarly Differential Diagnosis Crohn disease Infectious colitis Ischemic colitis in
elderly patients Radiation colitis Pseudomembranous Colon adenocarcinoma Rectal cancer Discussion
Background inflammatory disease, affecting the entire large Ulcerative colitis is a chronic idiopathic
primarily the rectum (95%). It has a worldwide distribution but is most common in northern Europe, and
Australia. age of 15 to 25 and a second at 55 to 65, although it may affected than men. The inflammatory
process in ulcerative colitis typically starts from the rectum, bowel but North America Its incidence is
higher among Caucasians, with a first peak at the present at any age. Women are more spreads from the
distal to the proximal colon terminal ileum (this is more commonly seen in patients with pancolonic
ulcerative colitis). The main pathologic feature of ulcerative colitis is the limited involvement of the
mucosal and submucosal lay- ers of the diseased bowel loops. Although ulcerative then (no skip areas)
sometimes also involving the colitis presents typical clinical, histologic, and

307 VII GITRACT PERITONEAL CAVITY AND RETROPERITONELM radiologic features, it cannot as infectious
colitis and always be distinguished Crohn disease (5-10% of patients from other bowel inflammatory
diseases may demonstrate Crohn-like such findings) Clinical Findings Most patients present abdominal
pain and weight Laboratory tests minemia, hypokalemia, hypomagnesemia). spondylitis, uveitis,
episcleritis, erythema nodosum, pyoderma gangrenosum, and sclerosing c persistent diarrhea, urgency
of defecation, tenesmus, fever, anorexia are characteristic features of ulcerative colitis may reveal
anemia, thrombocytosis, and abnomal serum chemical analysis (hypoalbu- loss. Rectal bleeding and
bloody diarthea manifestations are arthritis, synovitis, ankylosing cholangitis Extracolonic Complications
TOxic megacolon is an episode of fulminant transmural.colitis teading to neurogenic loss of motor and
progressive segments) It represents the most life-threatening complication of ulcerative colitis, as
patients are usually septic and need to be hospitalized. Other complications are strictures and bowel
obstruction. Several studies have demonstrated that colorectal cancer is a prime risk with ulcerative
colitis; the risk correlates discase tone colonic latation 6 cm; transverse colon is usually the most
commonly involved factor for mortality in patients with the extent of the inflammatory process, the
duration of and the presence of colorectal mucosal dysplasia, a well-known premalignant condition
Etiology The etiology of ulcerative colitis is unknown, although several studies suggest that
environmental and genetic members of the same family as well as within northern areas of North
America and Europe. Some have shown that smoking is protective against developing is reversed in
Crohn disease factors may be involved; in fact, a more common occurrence has been described within
studies ulcerative colitis this relationship Imaging Findings CT has becoTe a major demonstrate mucosal
lesions fistulas) colitis because of its ability to (abscess and lymphadenopathy. It also can be easily
performed in the setting of acute colitis (Le., toxic findings of ulcerative diagnostic tool for the evaluation
peribowel inflammatory ulcerative changes, intraperitoneal complications when bowel perforation is
suspected. The most important megacolon) colitis are imaging .Symmetric continuous wall thickening
(average thickness is less than in Crohn Stratified appearance of the and outer (muscularis propria) layer)
attenuation of this intermediate layer is usually negative in ulcerative colitis) Prominent enhancement of
the diseased walls after intravenous injection of contrast disease) affected (target or halo sign) with an
inner (mucosa) bowel loops surrounding a hypodense middle layer (submucosa) (the medium best phase
(the intensity of enhancement correlates with the degree of inflammation) core or plaquelike lesions are
suspicious seen in the enteric (fibrosis) or malignant (apple Strictures either benign for malignancy) Fat
proliferation surrounding the diseased bowel loops

308 TEACHING ATLAS OF ABDOMINAL IMAGING Treatment Pharmacotherapy Elective surgery may be
performed in case of medical therapy failure, drug-related side effects or presence of colonic dysplasia or
malignancy. Emergency colectomy may be required in the setting of fulminant colitis unresponsive to
medical aims to reduce and control the inflammatory process but is not curative. therapy Prognosis
Ulcerative colitis is characterized by periodic remissions and exacerbations, which may trolled by medical
therapy. Colectomy is curative. Long-term morbidity results from side effects of drugs (especially
steroids). be con- PLARLS (95%), then extends from the distal to the Ulcerative colitis almost always
affects the rectum proximal colon (no skip areas). Wall thickening is symmetric and less prominent than
that seen in Crohn disease Careful surveillance is needed for malignancy (risk is 5-30 times higher than
the general population) PITFALL Patients may not be possible to differentiate ulcerative colitis from
Crohn disease. present overlapping radiologic signs of inflammatory bowel disease, so that it may
Readings Suggested Desai RK, Tagliabue JR, Wegryn SA, Einstein DM. CT evaluation of wall thickening in
the alimentary tract. Radiographics Freeny PC. Crohn's disease and ulcerative colitis: evaluation with
double-contrast barium examina- tion and endoscopy 1991:11:771-784 Postgrad Med 1986:80(3):139-
146, 149, 152-1561 Giardiello FM, Bayless TM. Colorectal cancer and ulcerative colitis, Radiology
1996;199:28-30 Hortan KM, Corl FM, Fishman EK. CT evaluation of the colon: inflammatory disease.
Radiographics 2000:20:399-418 Kelvin FM, Oddson TA, Rice RP. Garbutt IT, Bradenham BP. Double
contrast barium enema in Crohn's disease and ulcerative colitis. AJR Am J Roentgenol 1978;131(2):207-
213 Paulsen SR, Huprich JE. Fletcher JG, et al. CT enterography as a diagnostic tool in evaluating small
bowel disorders: review of clinical experience with over 700 cases. Radiographics 2006:26:641-657
Philpotts LE, Heiken JP. Westcott MA. Gore RM. Colitis: use of CT findings in differential diagnosis
Radiology 1994:190:445-449 Roggeveen MJ. Tismenetsky M, Shapiro R. Best 2006:26:947-951 Thoeni RF,
Cello JP. CT imaging of colitis. Radiology 2006;240:623-638 cases from the AFIP: ulcerative colitis,
Radiographics

* CASE 95 Clinical Presentation A 64-year-old man complains of bloody diarrhea. Fig, 95.1 Axial T2-
weighted MRi image of the pelves shows ll-defined circumferential thickening of the rectum due to a low
signal i the turnor into the perirectal fat (arrowhead), T3 disease, as well as an adjacent metastatic
(curved arrow). intensity tumor (arrow). There is extension of making it a lymph node Radiologic
Findings Axial T2-weighted magnetic resonance (MR) image of the pelvis shows ill-defined
circumferential thickening of the rectum. There is soft tissue extension into the perirectal fat as well as
an adjacent lymph node (Fig. 95.1) Diagnosis Rectal adenocarcinoma with perirectal. extension of tumor
Differential Diagnosis Rectal lymphoma metastases (rare) Rectal Discussion Background Colorectal
carcinoma is the second most common occur in the rectum. Ninety- cancer in the Lnited States
Approximately one -eight percent of rectal cancers are adenocarcino third of these cancers they usually
arise from adenomatous polyps mas, and The Duke system and the TNM (tumor-nodess-metastasis)
system use the depth of tumor penetra- tion of the bowel wall and the presence of lymph node
involvement and disease. Accurate staging with preoperative imaging allows the most appropriate
treatment metastases to stage the option 309

with T1 disease are usually treated with local resection, those with T2 ar T3 and those with T4 disease
may be suitable 310 TEACHING ATLAS OF AlRDOMINAL IMAGING disease are usually treated with total
mesorectal excision, for preoperative chemoradiation in an attempt to downsize the tumor prior to
surgery to be selected. Patients weight Patients may present with a change in bowel habit, blood or
mucus per loss, and abdominal discomfort Findings rectum, tenesmus, Clinical Complications
Complications are bowel to the liver, lungs, and adrenals obstruction and perforation. Rectal carcinoma
most commonly metastasizes Etiology Risk factors include a high-fat/low-fiber diet, a family history of
colorectal carcinoma, a personal inflammatory bowel history of colorectal carcinoma or colonic polyps,
smoking, alcohol, diabetes disease, familial adenomatous polyposis, and Rectal carcinoma is more
hereditary nonpolyposis colorectal cancer (HNPCC) common in men than women and most commonly
occurs after age 50 Imaging Findings Endoluminal determine the T stage of the nodes in the adjacent
Computed tomography (CT) cannot clearly a sensitivity of only 79% for detection of extension of the
tumor through the bowel wall with adjacent in the detection of distant metastases MRI may be
performed using an endorectal allow clear delineation of the layers of the rectal wall; a wider field of
view and a clearer depiction of the mesorectal fat and fascia. have an accuracy of T staging of-86% tion is
paid to the distance of the tumor from the mesorectal tumor is of intermediate signal ulträsound clearly
delineates the layers of the wall of the rectum and can accurately enlarged lymph tumor in -85% of
cases. It may also demonstrate mesorectal fat and invasion of adjacent pelvic organs, demonstrate the
layers of the bowel wall and has tissue invasion. It is also poor at detecting involved adjacent lymph
nodes. CT is helpful coil or a surface phased aTray coil, Both however, surface phased array coils provide
techniques MRI is reported to Multiplanar imaging is performed, and particular atten- fascia. On T2-
weighted images, the perirectal fat and rectal wall. intensity, relative to the adjacent Prognosis The
prognosis depends on the stage of the 72% with TI and T2 disease and 7% with T4 disease. disease at
diagnosis with a 5-year survival rate of PEARL Contrast-enhanced T1 imaging has not been resolution T2-
weighted imaging is generally sufficient for diagnosis. shown to improve diagnostic accuracy. and high-

VI GI TRACT, PERITONEAL CAVITY, AND RETROPERITONELIM 311 PITFALL Endoluminal ultrasound, CT. and
MRL are poor at detecting regional nodal disease: this is partly because there are no agreed size criteria
for involved lymph nodes. New MRI contrast ultra-small particle jon-based contrast agents may improve
the characterization of regional lymph nodes. Suggested Readings Bipat S, Glas AS, Slors Fl, Zwinderman
assessment of lymph node involvement Radiology PM, Stoker 1. Rectal cancer: local with endoluminal
US, CT, and MR imaging-a meta-analysis AH, Bossuyt staging and 2004:232:773-783 Heneghan JP. Salem
RR, Lange RC, Taylor KI. Hammers IW, Transrectal sonography in staging rectal carcinoma: the role of
gray-scale, color-flow, and Doppler imaging analysis. AIR Am 1 Roentgenol 1997:169:1247-1252 lafrate F,
Laghi A. Paolantonio P et al. Preoperative staging of rectal cancer with MR imagingt cor- relation with
surgical and histopathologic findings, Radiographics 2006:26:701-714

* CASE 96 Clinical Presentation and weight loss A 52-year-old man complains of vague abdominal pain A
Fig. 96.1 Contrast enhanced axial CT images show circumferential thickening of the sigmoid colon (white
arrow) asso- ciated with adjacent pericolic lymph nodes (block arrow). There are no signs of bowel
obstruction Radiologic Findings Contrast enhanced axial computed tomography (CT) images
show.circumferential thickening of the sigmoid colon associated with adjacent pericolic lymph nodes.
There are no signs of bowel obstruc- tion (Fig. 96.1 Diagnosis Lymphoma of the sigmoid colon
Differential Diagnosis Adenocarcinoma of the sigmoid Metastases Infection and inflammatory bowel
Focal bowel ischemia (less likely) disease (less likely) Discussion Background Most primary
gastrointestinal (GI) tract lymphomas are non-Hodgkin lymphomas (NHLS). They are seen more
frequently in the stomach and small bowel than in the colon. They occur more in men than in women
commonly with a median age at diagnosis of 55 years. Primary colonic lymphoma is 312

VII GI TRACT, PERITONEAL CAVITY, AND RETROPERITONEUM 313 rare, accounting for 4% of cases of
extranodal NHL and < 1% of all colonic malignancies. The cecum and rectum are the most common sites
of involvement. Clinical Findings Patients with primary colonic lymphoma changes in bowel such as
usually present habit, abdominal pain, blood per rectum, and weight loss with nonspecific GI symptoms
Complications Complications include bowel wall perforation or ulceration, which can lead to massive
hemorrhage intra-abdominal Intussusception is a complication of cecal lymphoma. Colonic obstruction
rarely occurs Etiology Risk factors for the immunodeficiency virus development of primary colonic
lymphoma are immunosuppression human (HIV) infection, and inflammatory bawel disease Imaging
Findings Batium cnema in primary lymphoma of the cecum may Double-contrast imaging in cases of
primary Iymphoma of other areas of the colon may strate an infiltrating may be smooth or demonstrate
demonstrate a bullky polypoid mass demon- lesion, with some luminal narrowing. The mucosa
ulceration with thickening of the folds. Cavitation of the mass mass, seen as an annular may be seen
Segmental narrowing of the colon secondary to a circumferential soft tissue mass may be seen with CT
imaging The mass may node enlargement may be demonstrated. appear bulky and may demonstrate
cavitation Regional lymph Prognosis colonic lymphoma has a poorer prognosis than other forms of GI
NHL , with a 5-year survival Primary rate postsurgical resection and adjuvant chemotherapy of 27 to 55%
The prognosis depends on the depth of the wall invasion and the presence of involved lymph nodes or
disseminated disease. PEARL segment, obstruction propria and luninal narrowing of a bowel Although
lymphoma may lead to significant lymphoma infiltration leads to weakening of the muscularis does not is
rare because elicit a desmoplastic reaction PITEALL Patients in whom findings that differ nodules
involving a focal segment or distributed there is colonic involvement as part of generalized NHL may have
from those of primary colonic, imaging lymphoma. Multiple small. smooth submucosal throughout the
colon are generally seen

* CASE 97 Clinical Presentation left upper abdominal pain and fever episodes are A 45-year-old Armenian
man presents with 2 days of severe He has had similar episodes every associated with nausea and
vomiting, but tast a few days and are self-limited. His past medical history is otherwise significant for
remote appendectomy and small bowel obstruction with exploratory laparotomy and Iysis of adhesions.
few months since childbood. Sometimes these there is no associated diarrhea. The episodes usually fig
97.1 (A) Axial and cororial CT at the tine d initial presentation shows perico donic and perierteric soft
tissue inflamma ry stranding in the left upper abdomen. ( CT performed 2 mooths later shows teric
vascular engorgement. omental ind mesenteric edema, and fluid in the left Ustacle gutter fcontinued)
315

316 TEACHING ATLAS OF ARDOMINALIMACING foontinued) (C) A third CT. Fig. 97.1 performed 2 months
after the second cas shows nght nephromegaly and new aym metric right perirenal fat stranding: a 4 min
distal right ureteral stone was diagn shown). Notice the absence of infan (not matory changes in the
peritoneal cavity Radiologic Findings Axial and corbnal computed tomography (CT) scans at the time of
show pericolonic and perienteric soft tissue inflammatory stranding in the left performed 2 months later
(Fig. 97.18) shows mesenteric teric edema, and ond Fig. 97.1C) image a 4 mm distal right ureteral stone
changes in the peritoneal (Fig. 97.1A) upper abdomen CT initial presentation mesen- vascular
engorgement, omental and fluid in the left paracolic gutter. A third CT scan performed 2 months after
the sec- nephromegaly and new asymmetric right perirenal fat stranding; was diagnosed (not shown).
Notice the absence of inflammatory shows right cavity Diagnosis Acute abdominal attack in familia
Mediterranean fever Differential Diagnosis Enterocolitis Infectious colitis Inflammatory colitis Ischemic
colitis Acute appendicitis 5mall bowel obstruction Cholecystitis Renal colic Spontaneous bacterial
peritonitis Discussion Background Familial Mediteranean fever (FMF) is an autosomal recessive disease
prevalent in ethnic groups Mediterranean area, mainly in Sephardic nating in the nosis of FMF is usually
clinical because there are no specific imaging or laboratory tests, other than a molecular genetic test that
is not widely origi- The diag Jews, Arabs, Turks, and Armenians. available. It is usually a diagnosis of
exclusion.

317 VI CTRACT, PERITONEAL CAVITY, AND RETROPERITONEUM Clinical Findings IMF represents repeated,
self-limited episodes of fever rfaces, mainly of the peritoneum and joints. In 90% of cases. and
inflammation of synovial and serous the first attack occurs before age 20. Patients usually experience no
symptoms between attacks An acute abdominal attack usually presents with severe, diffuse abdominal
as an "acute abdomen." pain, rigidity, and fever, a clinical setting commonly defined Complications
Complications of repeated obstruction secondary to repeated peritonitis and previous explaratory
laparotomes complication of repeated attacks of FMF is the development of amyloidosis, which is
responsible for abdominal adhesive small bowel The dreaded attacks in patients with FMF include
increased morbidity and eventual mortality of some patients. reported as high as 60% in certain
populations with The prevalence of amyloidosis has been FMF, such as Turks Imaging Imaging findings on
computed teric folds Changes related to focal peritonitis may be visible Some patients may demonstrate
spienomegaly If there has been an adhesive small on imaging. Findings tomography (CT) are engorged
mesenteric vessels, thick mesen- and mesenteric and retroperitoneal lymphadenopathy. Ascites is a
frequent finding bowel obstruction, dilated small bowel loops will be visible Treatment includes
conservative therapy to relieve pain and inflammation and intravenous hydra- basis as a preventive
therapy. In the past, patients under- appendicitis and and the consider- Treatment tion. Colchicine may
be used on a daily went exploratory laparotomy to exclude surgical emergencies, such as acute adhesive
small bowel obstruction, although the era of cross-sectional imaging ation of the possibility of this
disease process in the proper setting may help prevent unnecessary surgery PEARL Radiologists should
be familiar with CT findings of peritoneal irritation in patients during an acute attack and may suggest
this clinical diagnosis in the proper patients who have not been previously with FMF clinical: setting in
diagnosed PITFALL During clinical evaluation, it may be difficult to differentiate an attack of FME from
otham abdominal conditions obstruction or, as in this case, urolithiasis, In these circumstances. directing
management. One would not want to assume that a patient's presentation with an acute. abdomen is
due to the known of the spectrum of causes of abdominal pain. acute appendicitis and small imaging
studies are useful for necessitating interventions, such as acute: bowel FMF as patients with FMF are
also susceptible to the remainder

*CASE 98 Clinical Presentation A 59-year-old woman with a history of breast cancer presents with a
complaint of abdominal pain and weight loss Fig. 98.1 (A,B)j Upper gastrointrstinal (Gl) study shows a
lobutated filling defect in the duodenal bulb (errow) There is no wall ircegulanty. (C) CT scan performied
n the same patient shows multiple rim-enhancing iesion with central in the fiver, likely areas of necrosis,
There is an enhancing mass in the antrum of the stomach (orrow). Given the appearance on the upper Gl
series and the CT appearance, the pri mary lesion is likely a pedunculated lesion arising from the antrum
and herriating into the duodenal bub reptesenting metastas Radiologic Findings (GI) study shows a
lobulated filling performed in the same patient defect in the duodenal gastrointestinal tomography (C1)
scan liver, likely antrum of the stomach (Fig. 98.1) bulb, Computed shows multiple rim-enhancing lesions
in the representing metastasis with central areas of necrosis. There is an enhancing mass in the Upper
Diagnosis gastric carcinoid type IIl (sporadic subtype) Pedunculated 319

IMAGING 320 TEACHING ATLAS OF ABDOMINAL Differential Diagnosis Adenocarcinoma of the gastric
antrum tymphoma GI stromal tumor (GIST) disease Metastatic polyp Giant pedunculated hyperplastic
Discussion Carcinoids are relatively uncammon neoplasms. of which -9% occur in the stomach, Gastric
car- cinoids are relatively less common malignancies stomach Zollinger-Ellison syndrome (ZES) and can
be seen in patients with multiple endocrine (MEN) type 1, and type lIll occurs sporadically. Type I is the
most common in the stomach. Non-ECL gastric carcinoids are rare Background Three subtypes of
enterochromaffin-like cell (ECL) carcinoids are recognized in the Type I is associated with autoimmune
chronic atrophic gastritis, type II is associated with neoplasia than other gastric neoplasms, representing
-1.8% of all gastric Clinical Findings Type I carcinoids are more frequent in women, with an average age
of 60. They are found incidentally during endoscopy performed for dyspepsia or anemia related to
chronic gastritis. Patients do not present with symptoms directly related to the carcinoid tumor. Type Il
carcinoids present equally, in both genders,, with a mean age of 50. Their pathogenesis is related to
elevated levels of gastrin-stimulating ECL hyperplasia. In the setting of MEN type 1, multiple carcinoids
can develop. lated that altered tumor suppression in MEN type 1 contributes to carcinoid tumor
formation Type III carcinoids occur more commonly in men, with an average age of 55. They are seen
more commonly in the gastric spread is present in 50 to 70% of cases generally Given that carcinoids are
rarely seen in patients with isolated ZES, it is postu- body and fundus They often demonstrate aggressive
features, and metastatic Complications In the absence of hepatic metastasis carcinoid syndrome does
not occur Imaging Findings Gastric carcinoids associated with chronic atrophic gastritis sized (1-2 cm)
mural masses: fluids may cause and ZES/MEN type 1 often well-marginated . Hypersecretions of gastric
Type II carcinoids may present as numerous nodules with a diffusely thickened There may be multiple
erosions and ulcers nodular mucosal and mural masses may be present. Gastric distention with neu-
media and scanning in the arterial and portal venous phase of contrast enhancement as multiple, and
fundus present located in the gastric body flocculations of barium and poor gastric variably mucosal
coating gastric wall On CT, numerous tral contrast may maximize detection of the prmary gastric lesion
and possible hepatic metastasis. Differential considerations for type I and 1 include entities that produce
multiple familial disease, and adenomatous polyposis, Peutz-Jeghers syndrome, Cronkhite-Canada
syndrome, Cowden multiple polypoid masses, as seen in hyperplastic polyps: Kaposi sarcoma; and
metastasis

321 v GITRACT, PERITONEAL CAVITY AND RETROPERITONELM Type III gastric fundus perigastric
adenopathy as well as hepatic metastasis Somatostatin-receptor scintigraphy is the study of choice in the
management of patients with carcinoids often masses in the body or present as large ( 2 cm) solitary
mural Given the malignant potential, patients should be screened for They may be ulcerated carcinoid
tumors, Indium In 111 pentetreotide is the most the primary tumor or stage the disease. Positron
effective due to slow tumor agent, often to localize commonly used emission tomography has not been
shown to be growth and low .cellular turnover Treatment and Prognosis Type I gastric carcinoids are
generally benign. Patients can be treated conservatively withe scopic follow-up or partial Type 11 gastric
endo- . gastric resection Metastasis occurs in < 2% of patients Carcinoids are the least common and are
prone to lymphatic metastasis, Tumor related deaths and carcinoid syndrome are rare. Treatment is not
uniform. Gastrectomy, polyp- ectomy . Type III is the most absence of hepatic metastasis, total
gastrectomy with en bloc and treatment with somatostatin analogues have been reported. and-20%
survival in 5 years. In the resection of lymph nodes is recom aggressive subtype. with a poor prognosis
mended. Chemotherapy and antitumaral therapies are recommended when hepatic metastases are
present PEARL AND PETFALL GI carcinoids have a diverse spectrum of clinical, pathologic, and radiologic
appearances on their origin based within the GI tract Suggested Readings Binstock Al. Johnson CD,
Stephens DH. Lloyd RV, Fletcher JG. Carcinoid tumors of the clinical and radiographic study. AIR Am J
Roentgenol 2001;176:947-951 Halpert RD. Gastrointestinal imaging: the requisites. 3rd ed. St. Louis: CV
Mosby: 2006 Levy AD, Sobin LH. From the archives of the AFIP Gastrointestinal carcinaids
clinicopathologic comparison. stomach: a imaging features with Radiographics 2007:27:237-257

* CASE 99 Clinical Presentation Nonspecific abdominal pain Fig. 99.1 Upper gastrointestinal series shows
an outpouching (aow) arising from the gastric fundus Radiologic Findings Upper gastrointestinal (GI)
series shows an outpouching arising from the gastric fundus (Fig. 99.1) Diagnosis Gastric fundal
diverticulum Differential Diagnosis Cystic adrenal, renal, or paricreatic lesions Necrotic lesions or abscess
ases where air-luid levels are present Duplication cysts Discussion Background Fundal diverticula are the
most common gastric diverticula are congenital and usually seen on the posterior wall of the stomach.
These are true diverticula and contain all layers of the stomach wall: they are vatiable in size. ranging
from 1 to 10 cm. arising from the cardiofundic region. They 322

Vil GI TRACT, PERITONEAL CAVITY, AND RETROPERITONELIM 323 Flo. 99.2 Axial CT image of the upper
abdomen shows a contrast-filled fundal diverticulum (arrow) Clinical Findings Patients are often
asymptomatic: Some patients may present with intermittent pain, worse when supine Complications
Usually these are asymptomatic: however, cases of bleeding and ulcerations exist. Patients may pre- sent
with pain Additionally, the condition may as an adrenal mass and abdominal discomfort. If diverticula are
large, impaired emptying may lead to excessive work-up due to misinterpretation of a diverticulum occur
Imaging Findings Lateral view of a barium swallow and upper Gl studies may demonstrate an
outpouching on the posterior aspect of the tomography gastric cardia, with a characteristic finding of a
fundal diverticulum. arising from the posterior may show a collection of contrast material aspect of the
gastric fundus. It usually projects in the retroperitoneal cavity Occasionally, when Computed poorly
opacified with oral, considered in the differential for an adrenal pseudomass contrast, a fundal
diverticulum may mimic an adrenal mass and is (Fig 99.2) Treatment and Prognosis Treatment is
conservative and expectant. . PEARL Classic barium studies appearance on

324 TEACHING ATLAS OF ARDOMINAL IMAGING PITEALL When poorly filled with oral contrast or if
collapsed, gastric fundal diverticula may mimica lesions adrenal Suggested Readings Bothen NE Eklof O.
Diverticula and dunlications (enterogenous cysts) of the stomach and duodenum Am Jenten Ruclol
1966: 96: 375-381 Halpert RD. Gastrointestinal Imaging: The Requisites 3rd ed. St. Louis: CV Mosby:
2006 schwartz AN, Goiney RC, Graney DO, Gastric diverticulum simulating an adrenal mass: CT
appearance and embryogenesis. AIR Am J Roentgenol 1986: 146: 553-554

*CASE 100 Clinical Presentation A middle aged man with a recent history of a high-speed motor vehicle
accident who underwent emergent open reduction and internal fixattion of pelvic fractures presents
months later with pelyic pain and fevers Fig. 100.1 (A) obscured by the orthopedic hardware is CT images
show a right pelvic vind around a spongiform gas collection with mass effect on the rectum (D) The
radiodense ribbon is best seen on bone windows and hs within the collection (curved arrow) Supine
abdominal radiograph shows screw and plate fixations of the patient's pelvic fractures. Partially
radiodense ribbon at the level of the pubis (orrow). (B-D) Contrast-enhanced space that demonstrates an
enhancing abscess collection (arrow) in the right obturator Radiologic Findings Supine abdominal
radiograph 100.1A). Partially obscured by the orthopedic hardware is a radiodense ribbon at the level of
the pubis. Contrast-enhanced in the right obturator with mass effect on the rectum. The radiodense
ribbon is best the collection (Fig. 100.18-D shows screw and plate fixations of the patient's pelvic
fractures (Fig. pelvic abscess collection computed tomography (CT) images show a right :3pace that
demonstrates an enhancing rind around a spongiform gas collection seen on bone windows and is within
325

IMAGING 326 ACHING ATLAS OF ARDOMINAL Diagnosis Gossypiboma (textiloma or retained surgical
sponge Differential Diagnosis Abscess as a complication of Acute appendicitis diverticulitis Acute Surgery
Perforated tumor or stercoral ulcer Tumor Hematoma Discussion 8ackground hemorrhage and other
Laparotomy sponges are used to pack the exposed laparotomy wound to absorb body fluids and to help
with hemostasis The most common retained foreign body after surgery is the cotton laparotomy A
retained surgical sponge is known as a gossypiboma, from the Latin gossypium for "cotton" and the
Swahili homa fon pad, a rectangular sponge with an integrated radiopaque tag, as is seen in this case
"place of concealment" Retention of surgical swabs or sponges occurs in 1 in 100 to 5000 operations in
the United States and is an important cause of malpractice claims Clinical Findings A retained sponge
may be asymptomatic or may present with evidence of sepsis and abscess forma- tion or gastrointestinal
symptoms a delayed manner such as bowel obstruction. Symptoms may present acutely or in
Complications A retained sponge can lead to an aseptic granuloma with a surrounding fibrinous
response that may be asymptomatic or to a more morbid exudative response with complications, such as
abscess forma- tion and sepsis, intraperitoneal adhesions, and fistula formation. Extrusion into the
hollow viscera has been reported Etiology careful attention by surgical teams, sponges are, on occasion,
left in the abdomen and pelvis Despite A normal sponge count does not exclude the presence of a
retained sponge. Imaging Findings Linear, spongiform, or whirl-like radiolucencies reflect gas trapped in
the folds of the retained tabric on radiographs or CT; the bady of the sponge itself is usually not detected
l sponges or swabs are labeled with radiopaque markers that will make them e Tags, as in this case, may
appear crenulated when inside the patient. Calcifications may Many surgical to detect form in chronic
collections around surgical sponges easier

327 V CI TRACT, PERITONEAL CAVITY, AND RETROPERITONEUM On contrast-enhanced CT there is usually


dense rim enhancement complex mass in a patient with a history of surgery On ultrasound, retained
surrounding a low-density or pnges are nonspecific in appearance and may be seen as complex col- or
lections or as echogenic.and densely shadowing masses even in the albsence of retained p calcifications
On magnetic resonance imaging, tetained sponges have a variable appearance but usually resem- have
ble a soft tissue mass with a well-defined T1 and T2 hypointense enhancing capsule They a low-signal
internal whorled appearance on T2 sequences Treatment Exploratory of retained laparotomy and
removal of the retained surgical sponge are indicated. Complications sponges require appropriate
management. PEARLS Radiologists should be careful to investigate the presence of a surgical sponge on
imaging. because too frequently familiar with the imaging appearance of sponges used at their
institutions Radiopaque tags on retained sponges are often most easily seen on scout radiographs for CT
scans due to beam hardening artifacts on the axial images should be sponges are inadvertently left
inside patients. In addition, radiologists PITFALLS On occasion sponges are intentionally left in a patient
when a second-look Absorbable hemostatic operation is planned sponges can mimic the appearance of
an abscess. Suggested Readings Hunter TB, Taljanovic 503-523 MS. Medical devices of the abdomen and
pelvis. Radiographics 2005:25: Kim CK, Park BK, Ha H. Gossypiboma in abdomen and pelvis: MRI findings
in four patients, AJR AIm J Roentgenol 2007:189:814-817 Kokubo T. Itai Y, Ohtomo K, Yoshikawa K, lio M,
Atomi Y. Retainedi ance. Radiology 1987:165:415-418 O'Connor AR pelvis. AJR Am J Roentgenol
2003:180:481-489 surgical sponges: CT and US appear- Coakley FV, Meng MV, Eberhardt S. Retained
surgical sponges in the abdomen and

* CASE 101 Clinical Presentation A middle-aged patient complains of a palpable abdominal wall mass Fig.
101.1 Axial noncontrast CT image through the pelvis demonstrates a defect in the anterior abdominal i
wall, just lateral to the semilunar line. The sigmoid colon is seen herniating through this defect (arrow).
Radiologic Findings demonstrates a defect in Axial noncontrast computed tomography (CT) image
through the pelvis the anterior through this defect (Fig. 101.1) abdominal wall, just lateral to the
semilunar line. The sigmoid colon is seen herniating Diagnosis Spigelian hemia Differential Diagnosis
Incisional hernia Discussion Background Soicelian hernias are ventral hernias that occuT at the
intersection of the semilunar line and semicir- cular line. The lateral edge of the rectus muscles forms the
semilunar line; the semicircular line is the point inferior which only the transversalis fascia is posterior to
the rectus abdominalis muscle. Clinical Findings Spigelian hernias are well described in the literature but
occur rarely, They can be difficult to diag- nose clinically because the herniated contents can remain
between the lavers of muscle and fascia of the anterior abdominal wall 328

329 Vi GI TRACT, PERITONEAL CAVITY, AND RETROPERITONEUM Complications Incarceration leading to


infarction can occur Etiology The lack of a posterior sheath in this location creates an area of relative
weakness Treatment Surgical repair of the fascial defect is required once the diagnosis is made, as up to
20% will be incarcerated at diagnosis Repair does not require prosthetic . mesh, although it is being used
by some now performing laparoscopic repair. Prognosis Surgical repair has a good prognosis. PEARL The
location of this hernia is classic.for spigelian hernias. Suggested Readings Harrison L, Keesling CA. Martin
NL Lee KR. Wetzel 1.H. Abdominal wall hernias: review of hermiog- raphy and correlation with cross-
sectional imaging Palanivelu C. Vijaykumar M. Jani KV, Rajan PS, Maheshkumaar GS, Rajapandian S.
Laparoscopic trans- abdominal Surgeons 2006;10:193-198 Radiographics 1995:15:315-332 preperitoneal
repair of spigelian hernias. JSLS Journal of the Society of Laparoendoscopic

*CASE 102 Clinical Presentation A 42-year-old woman presents with abdominal pain and fever 10 days
atter laparoscopic chole ystectomy for severe gangrenous cholecystitis with a gallbladder full of stones
Fig. 102.1 (A-C) Axial CT images with oral and intra- venaus contrast demonstrate cholecystectomy clips
Caudal images show a subhepatic fluid collection with rim enhancement (anow) and surrounding fat
stranding containing four radiopaque gallstones (curved arow) Radiologic Findings Axial computed
tomography (CT) images with oral and intravenous contrast demonstrate a subhe- patic fluid collection
with rim enhancement and surrounding fat stranding containing four radi- apaque gallstones Fig 102.1
Diagnosis Subhepatic ahscess secondary to dropped gallstones 330

331 VII CITRACT, PERITONEAL CAVITYy, AND RETROPERITONEUM Differential Diagnosis Simple abscess .
Tumor Hematoma Biloma Discussion Background Spillage of gallstones during laparoscopic
cholecystectomy occurs in-4 to 50% of cases. Complications from dropped stones are very rare and occur
in only -0.08 to 0.30% of patients. Clinical Findings Clinical presentation is usually related to signs and
symptoms of abdominal pain and infection Patients may present as soon as 1 month to as long as 20
years after laparoscopic cholecystectomy Complications Complications of laparoscopic cholecystectomy
include bile duct injury and bile and/or gall- stone spillage. Late infections and abscess formation are
uncommon complications of dropped gallstones. Etiology Spilage of gallstones may occur during the
dissection of the gallbladder from the liver bed or during extraction of the gallbladder from the
abdominal wall port sites, A combination of multiple gallstones and infected bile increases the incidence
of abscess formation Imaging Findings Cross-sectional imaging, including ultrasound, CI, and magnetic
resonance, demonstrate an abscess collection containing gallstones or stone fragments generally
confined to the subhepatic space or the retroperitoneum below the subhepatic space. Unusual locations
have been reported, including the right thorax, the subphrenic space, the abdominal wall, and the site of
incisional hernias Treatment Treatment consists of percutaneous drainage of the abscess followed by
removal of the gallstones If complete evacuation of all gallstones cannot be obtained with minimally
invasive procedures. laparotomy may be necessary

TEACHING ATLAS OF ABDOMINAL IMAGING 332 Prognosis With complete evacuation of all of the
gallstones after abscess drainage, the prognosis is excel- lent. Any retained stones can be a nidus for
recurrent infection. PEARLS A high index of suspicion is paramount in the correct diagnosis of spilled
gallstone abscess Radiologists should consider this diagnosis in patients with intra-abdominal abscess
and a remote history of laparoscopic cholecystectomy. Identification of radiopaque stones within the
abscess cavity on CT is diagnostic of this condition. PITFALL Failure to recognize radiopaque dropped
gallstones in the abscess collection may lead to misdi- agnosis of a simple abscess or tumor, which may
lead to delay or inadequate treatment. Suggested Readings Horton M. Florence MG. Unusual abscess
patterns following dropped gallstones during laparoscopic cholecystectomy. Am J Surg 1998:175:375-379
Morrin MM, Kruskal JB, Hochman MG, Saldinger PF, Kane RA. Radiologic features of complications arising
from dropped gallstones in laparoscopic cholecystectomy. AJR Am J Roentgenol 2000;174: 1441-1445
Rice DC, Memon MA, Jamison RL, et al. Long-term consequences of intraope gallstones during
laparoscopic cholecystectomy.J Gastrointest Surg 1997erative spillage of bile and 1:85-90 Sathesh-Kumar
T, Saklani AP, Vinayagam R, Blackett RL. Spilled gallstone during laparoscopic chole- cystectomy: a review
of the literature. Postgrad Med J 2004:80:77-79 Treotola SO, Lillemoe KD, Malloy PC, Osterman FA.
Percutaneous removal of "dropped" gallstones after laparoscopic cholecystectomy. Radiology
1993:188:419-421
* CASE 103 Clinical Presentation A 73-year-old woman presents with abdominal discomfort and flank
pain Fig. 103.1 Coronal reformatted contrast-enhanced CT image show a heterogeneous, ill-defined mass
in the left perinephric region surrounding the left kidney. The lesion shows areas of fat density and foci
of calcifications adjacent to the left kidney Radiologic Findings Coronal reformatted, contrast-enhanced
computed tomography (CT) image shows a heterogeneous, ill-defined mass in the left perinephric region
surrounding the left kidney. The lesion shows areas of fat density and foci of calcifications adjacent to
the left kidney (Fig. 103.1). Diagnosis Dedifferentiated retroperitoneal liposarcoma Differential Diagnosis
Malignant fibrous histiocytoma Retroperitoneal lymphoma Metastases Retroperitoneal dermoid
Infection (rare) Discussion Background Liposarcoma is the second most common type of soft tissue
sarcoma in adults and is most commonly seen in the 5th decade of life. 333

TEACHING ATLAS OF ABDOMINAL IMAGING 334 Clinical Findings Abdominal fullness Flank pain Vomiting
Abnormal renal function tests in patients with renal invasion Hematuria Complications Invasion of
adjacent organs, mainly kidneys Metastases (very rare) Etiology Unknown Imaging Findings On imaging,
a retroperitoneal, large, ill-defined, heterogeneous, fat-containing mass with involve- ment of adjacent
organs and bridging across the fascial planes is seen. Tumors are almost always large in size at
presentation, and patients usually present with nonspecific symptoms. CT may show calcifications that
are commonly nodular in configuration. Involvement of adjacent organs, mesenteric vessels, and renal
vessels is common and should be carefully assessed at imaging Contiguity with an adjacent organ
remains an important issue for complete assessment because tumors tend to invade adjacent organs
without any direct evidence of involvement on imaging. The presence of fat on CT and magnetic
resonance imaging (MRI) helps in diagnosing the condi- tion: however, macroscopic fat is not always
present in liposarcomas (Fig. 103.2) Fig. 103.2 Axial contrast-enhanced CT image in a differ ent patient
shows a hypoattenuating lesion with a thick enhancing capsule. The lesion is posterior to the kidney
compressing and displacing the icidney anteriorly. Because no discrete focus of macroscopic fat was
seen, It was dif- ficult to diagnose this lesion as liposarcoma; however, the lesion was found to be
dedifferentiated liposarcoma on sur- gical pathology.

Vi GITRACT, PERITONEAL CAVITY, AND RETROPERITONEUM 33 Treatment Surgical resection of the tumor
is the treatment of choice in patients without metastases Contiguity with the kidneys necessitates
nephrectomy in addition to resection of the primary neoplasm Prognosis The prognosis is worse in
patients with renal involvement and vascular encasement of the aorta and/or inferior vena cava.
However, in patients with surgically resectable lipasarcoma without adjacent organ involvement, the
prognosis remains excellent after treatment, although long-term follow-up imaging is important to
detect recurrence PEARL Large, ill-defined, heterogeneous, fat-density retroperitoneal lesion PITFALL
Lesions that do not demonstrate macroscopic fat on MRI and fat density on CT cannot be dif- ferentiated
from other retroperitoneal sarcomatous lesions. necessitating a biopsy. Suggested Readings Gupta AK,
Cohan RH, Francis IR, Sondak VK, 1Korobkin M. CT of recurrent retroperitoneal sarcomas AIR Am J
Roentgenol 2000;174:1025-1030 Nishino M, Hayakawa K. Minami M. et al. CT and MR imaging findings
with anatomic and pathologic diagnostic clues. Radiographics 2003:23:45-57
*CASE 104 Clinical Presentation A 91-year-old man presents with abdominal pain and nausea. Fig 104.1
(A) Axial CT image of the abdomen shows multiple contrast-filled, dilated small bowel loops and a small
mes- enteric mass (orrow) with a surrounding desmoplastic reaction and an adherent small bowel loop,
which demonstrates a circumferentially thickened wall and a narrowed lumen. (8-D) MR images of the
abdomen in the same patient demonstrate the small mesenteric mass that is isointense to muscle on T1
and T2, linear bands radiate from the mass and cause retrac tion of the adjacent small bowel imesentery
Radiologic Findings Computed tomography (CT) scan of the abdomen performed at the time of
presentation demon- strates multiple dilated small bowel loops and a small mesenteric mass with a
surrounding desmo- nlastic reaction and an adherent small bowel loop, which demonstrates a
circumferentially thickened wall and a narrowed lumen. Magnetic resonance imaging (MRI) of the
abdomen in the same patient demonstrates the small mesenteric mass as 1Sointense to muscle on T1
and T2: linear bands radiate from the mass and cause retraction of the adjacent small bowel mesentery
(Fig. 104.1). 336

33 Vl cI TIACT, PERITONEAL CAVITY, AND RETROPERITONEUM Diagnosis Small bowel carcinoid with
mesenteric metastases Differential Diagnosis Primary mesenterie desmoid tumor . Sclerosing
mesenteritis (fibrosing mesenteritis). . Metastatic disease from pancreatic, gastric, or colon cancer Other
primary gastrointestinal (GI) malignancy (eg, adenocarcinoma or Gl stromal tumor) Lymphoma .
Endometriotic implant (in a woman) Discussion Background Carcinoid tumors arise from neuroendocrine
cells anywhere in the body but most frequently occur in the GI tract or bronchopulmonary system. They
have variable potential for malignancy, and 20. of patients have metastases at the time of diagnosis,
Metastatic disease occurs in 300 to 50% of all patients and may occur at any time, from diagnosis up to
20 years after presentation. The most fre- quent sites of metastatic disease from a carcinoid arising from
the midgut are the liver and mesen- teric lymph nodes, Midgut carcinoid tumors most often spread to
the mesentery by direct extension, but they also occur by invasion of local lymphatic vessels. The
primary intestinal carcinoid is often stmall and difficult.to detect on CT, whereas a dominant mesenteric
mass may be discovered first Clinical Findings and Complications Many patients are asymptomatic at the
time of presentation. Other patients may present with nanspecific abdominal pain, a palpable abdominal
mass. diarrhea, small bleeding, Patients with liver metastases may experience carcinoid syndrome with
episodic flushing wheezing, and diarrhea, Elevated levels of serotonin or 5-hydroxytryptophan are found
in the blood. and 5-hydroxyindoleacetic acid (5-HIAA) levels are elevated in the urine owel obstruction,
or GI Etiology Neuroendocrine tumor arising from the enterochromaffin or Kulchitsky cells Imaging
Findings On CT, mesenteric metastases appear as ill-defined enhancing soft tissue masses Seventy
percent of mesenteric metastases contain calcifications (Fig. 104.21. Low-attenuating, nonenhancing foci
within the soft tissue mass may represent tumor necrosis and can produce a rimlike pattern of
enhancement on CT and MRI. Linear, "fingerlike" radiating bands may be seen in the mesenteric fat with
adjacent tethering of the stmall bowel. This repre sents a mesenteric desmoplastic reaction secondary to
fibrosis and mesenteric ischemia caused by the release of serotonin by the tumor (Fig. 104.3) CT
angiography reveals tethering, encasement, or occlusion of mesenteric vessels.
TEACHING ATLAS OF ABDOMINAL IMAGING 338 Fig. 104.3 Coronal reformatted CT image shows a soft
tissue mesenteric mass (orrow). There is accom panying desmoplastic reaction in the mesentery and
small bowel ischemic changes Fiq. 104.2 Axial contrast-enhanced CT image shows a solt tissue
mesenteric mass (anow) with calcifications Surrounding desmoplastic reaction is seen. MRI
demonstrates a mesenteric mass that is T1 and T2 isointense to muscle with hypointense desmoplastic
stranding Mesenteric metastases are strongly positive on octreotide scintigraphy. A fluoroscopic
examination or CT enteroclysis may show target lesions from ulceration of a sub- mucosal tumor,
thickening mucosal folds, and tethering of small bowel loops. Treatment A distal small bowel carcinoid
with mesenteric metastases may be treated by surgical resection of bowel and mesentery often with a
hemicolectomy. Proximal small bowel tumors are treated with pancreaticoduodenectomy. Many
palliative options are also available, including cytotoxic agents, somatostatin analogues, and
chemoembolization of liver metastases. Prognosis Carcinojd tumors are relatively slow-growing
neoplasms. Even in the presence of metastatic dis- ease, patients may survive many years. The overall 5-
year survival rate for all patients with a carcinoid tumor is -70 to 80% PEARL .Consider carcinoid with
mesenteric metastases for any saft tissue mass arising within the small bowel mesentery with
calcifications or a desmoplastic reaction.

VI GITRACT, PERITONEAL CAVITY, AND RETROPERSTONEUM 339 PITFALLS Mesenteric carcinoid


metastases may present as a large, dominant mesenteric mass, and a pri- mary tumor is often difficult to
identify at CT. A close examination of the small bowel mucosa is recommended to evaluate for the
presence of a small primary carcinoid tumor Retractile mesenteritis cannot be distinguished from a
carcinoid tumor or lymphoma by imaging alone, and biochemical analysis or histologic sampling is
required to make the distinction Suggested Readings Levy A, Sobin L From the archives of the AFIP
clinicopathologic comparison. Radiographics 200strointestinal carcinoids: imaging features with :27:237-
257 Scarsbrook A. Ganeshan A, Statham J, et al, Anatomic and functional imaging of metastatic carcinoin
tumors. Radiographics 2007:27:455-477 Sheth S, Horton K, GarlandM, Fishman E. Mesenteric
neoplasms: CT appearances of primary and secondary tumors and differential diagnosis. Radiographics
2003:23:457-473

*CASE 105 Clinical Presentation A 26-year-old man presents with a relatively short history of constipation
and urinary proble Fig. 105.1 (A8) Axial contrast-enhanced CT miges demonstrane a lange fow
attensuation mass with coarse central calcifica ts lecated anterior to the urinary bladder Asubtle cortical
delect is present in the left pubic symphysis where the mass abuts tiane), (C.D) Aal T1-weighted MR
images demonstrate a large T1 hypointemse mass causing mass effect on the urinary bladder (E) The
mass is heferogeneously bright on an auial T2 sequence and (F) demonstrates heterogeneous,
predominantly peripheral enhancement on the fab-suppressed T1 postoontrast image 340
341 vI GI TRACT PERITONEAL CAVITTY, AND RETROPERITONEUM Radiologic Findings An axial contrast-
enhanced computed tomography (CT) scan (Fig. 105.1A,B) demonstrates a large low-attenuation mass
with coarse central calcifications located anterior to the urinary bladder. A sub tle cortical defect is
presemt in the left pubic symphysis where the mass abuts it. Axial T1-weighted magnetic resonance
(MR) images (Fig. 105.1CD) demonstrate a large T1 hypointense mass caising mass effect on the urinary
bladder. The mass is heterogeneously bright an an axial 12 sequence and demonstrates heterogeneous,
predominantly peripheral enhancement on the tat-suppressed T1 pastcontrast image (Fig. 105.1EF)
Diagnosis Juxtacortical chondrosarcoma mimicking a pelvic visceral mass Differential Diagnosis Sarcoma
Malignant fibrous histiocytoma Poorly differentiated liposarcoma Leiomyosarcoma Bladder tumor
Urachal adenocarcinoma (mucinous type) Rhabdomyosarcoma Pheochromocytoma Leiomyomas
Hemangioma Periosteal osteosarcoma and other tumors of bony origin Discussion Background
Chondrosarcoma is the third most common primary malignant bone tumor About 4% of chondro-
sarcomas are juxtacortical chondrosarcomas that arise on the surface of bone and occur in adults in the
3rd to 4th decade. Because of a lack of symptoms, these lesions may be very large at the time of
diagnosis and may mimic a soft tissue mass. They generally contain a chondroid matrix and metaplastic
ossification. Clinical Findings Patients present with nonspecific symptoms and signs and most commonly
have a painless, paipable mass that is slow growing. Symptoms may be due to mass effect on adjacent
organs Etiology Chondrosarcomas may occur primarily or result from mailignant degeneration of
osteochondromas or enchondromas

342 TEACHING ATLAS OF ABDOMINAL IMAGING he most suggestive radiographic finding of


chondrosarcomas is a lytic lesion with chondroid matrix and mineralization in the form of rings and ares
Aggressive features such as endosteal scalloping, cortical destruction, and soft tissue extension are best
evaluated by CT scanning Chondrosarcomas have high water content and therefore appear as low
attenuation on CT and demonstrate high signal intensity on T2-weighted images on MRI. Enhancement is
seen in the septa, with a ring and are pattern Imaging Findings Treatment Treatment is surgical and
depends on the histologic grade of the tumor An aggressive, wide surgical excision is necessary for high-
grade tumors and chemotherapy are used as adjunctive treatment for higher grade chondrosarcomas
prevent local recurrence and metastasis. Radiation Prognosis The overall 5-year survival rates for
chondrosarcoma are 90 to 94% (grade 1), 61 to 81% ( grade 2), and 43 to 44 % ( grade 3 ) ,
Dedifferentiated chondrosarcomas have the worst prognosis. PEAR The presence of a chondroid matrix
with mineralization in rings and arcs is highly suggestive of a chondrosarcoma PITFALL Juxtacortical
chondrosarcomas may mimic a soft tissue mass because they occur on the surface of bones and are
usually very large at the time of diagnosis. Although malignant, chondrosarcomas may be well
circumscribed and appear benign, Suggested Readings Murphey MD, Wwalker EA, Wilson Al. Kransdorf
MI. Temple HT. Gannon FH. From the archives of the AFIP. Imaging of primary chondrosarcoma:
radiologic-pathologic correlation. Radiographics 2003:23:1245-1278 Varma DG. Ayaia AG, Carrasco CH,
Guo SQ, Kumar R. Edeiken J. Chondrosarcoma: MR imaging with pathologic correlation. Radiographics
1992:12:687-704
*CASE 106 Clinical Presentation A 34-year-old man is evaluated for chest pain. Fig. 106.1 (A-C) Axial CT
images show a lobulated, low attenuation mas, inferior to the body and tal of the pan- Cieas without
enhancement Radiologic Findings Axial computed tomography (CF) images reveal a lobulated, low-
attenuation mass, inferior to the body and tail of the pancreas without enhancement (Fig. 106.1
Diagnosis Retroperitoneal lymphangioma 343

344 TEACHING ATLAS OF ABDOMINAL IMAGING Differential Diagnosis Enteric duplication cyst Enteric
cyst Mesothelial cyst Pancreatic pseudocyst Cystic leiomyoma or leiomyosarcoma Discussion Background
Approximately 95% of lymphangiomas are found in the neck and axilla. Other locations include the
retroperitoneum, mesentery, omentum, mediastinum, and bone. Retroperitoneal lymphangiomas
represent 1% of retroperitoneal neoplasms. They tend to grow and insinuate between organs Clinical
Findings Retroperitoneal lymphangiomas are usually found in infancy because of symptoms, but they
may be found as asymptomatic lesions in adults. Symptoms when present vary but can include a
palpable mass, fever, nausea, and pain. Complications Hemorrhage Infection Displacement of the
kidneys Etiology Lymphangiomas are congenital lesions that develop when there is a failure in the
formation of nor- mal connections within the lymphatic system Imaging Findings On ultrasound,
lymphangiomas appear as anechoic, well-defined lesions with increased through- transmission. Internal
septa can be seen. On CT, lymphangiomas appear as multiloculated lesions with Hounsfield units ranging
from near water to near fat, depending on their composition On magnetic resonance imaging, they tend
be isointense to slightly hypointense to muscle on T1-weighted images and appear hyperintense to fat
on T2-weighted images. Treatment Surgical excision

Vil CI TRACT, PERITONEAL CAVITY, AND RETROPERITONEUM 345 Prognosis Complete surgical excision has
a lower recuerence cate than aspiration or injection of a selerustng agent. PEARL Lymphangiomas can
cross from one retroperitoneal compartment to another PITFALL Lymphangiomas can measure near
water density to near fat density Suggested Readings Davidson Al. Hartman DS. Iymphangioma of the
retroperitoneum: CT and sonographic characteristics Radiology 1990:175:507-510 Mar CR,
Pushpanathan C. Price D, Cramer B. Best cases from the AFIP: omental lymphagioma with a small bowel
volvulus. Radiographics 2003:23:847-851 Yang DM. Jung DH, Kim H, et al. and literature review.
Radiogranetroperitoneal cystic masses; CT, clinical and pathologic findings hics 2004:24:1353-13651

*CASE 107 Clinical Presentation A 46-year-old woman presents with a history of retroperitoneal
liposarcoma status postresection Fig. 107.1 (A) Axial and (B) coronal contrast-enhanced CT images
shhowa rounded, relatively homogeneaus mass that measures water density by Hounsfield units in the
retroperitoneal region adjacent to the ln uscle (arrow) Radiologic Findings Axial and coronal contrast-
enhanced compated tomography (CT) images show a rounded, relatively homogeneous mass that
measures water density by Hounsfield units in the retroperitoneal region adjacent to the left psoas
muscle (Fig. 107.1) Diagnosis Metastatic liposarcoma with pseudocystic sign Differential Diagnosis
Seroma Retroperitoneal lymphocele Abdominal abseess Necratic solid tumor Lymphangioma 346
Vi CI TRACT, PERITONEAL CAVITrY, AND RETROPERITONEUM 347 Discussion Background Liposarcoma is a
malignancy of adipose cells, It is the most common primary retroperitoneal tumor and the second most
common soft tissue sarcoma in adults. Liposarcomas make up 95% of all fattyy retroperitoneal tumors,
They are usually seen in patients ages 40 to 60 years and occur more com- manly in men than in women.
Clinical Findings and Complications Liposarcomas are usually asymptomatic until they become large.
Patients.may present with abdomi- nal pain due to impingement of neighboring structures by an
enlarging mass. The tumor may come to attention if the patient has an episode of trauma in the resion of
the mass Other symptonmis are painful swelling, decreased motion, nurmbness, and weight loss.
Etiology There is no well-established causative factor, although trauma has been implicated. Rarely,
iposar- comas arise from lipomas Imaging Findings Both CT and magnetic resonance imaging (MRI) can
be useful for imaging liposarcomas. CT is better at assessing cortical bone erosion and tumor
mineralization, whereas MRI is more effec- tive at depicting the fatry natiure of the tumo. Liposarcomas
that are a mixture of fat and tissue can appear to be water density on CE due to volume averaging,
commonly referred to as the pseudocystic pattern. Well-differentiated liposarcomas have a well-defined
or lobulated border and are composed mainly of adipose tissue. Well-differentiated liposarcomas may
show faint enhancement after intravenous contrast administration. As the grade of malignancy
increases, contrast enhancement becomes more intense and heterogeneous .The noncontrast
appearance of myxoid liposarcomas may be homogeneous or mildly heteroge- neous. However,
postcontrast, myxoid liposarcomas demonstrate moderate to marked hetero- geneous enhancement.
Pleomorphic liposarcomas are markedly heterogeneous both before and after contrast administration
Risk assessment can be dune based on 18F-fluorodeoxyglucose positron emission tomography imaging
for liposarcomas by determining the pretherapy standardized uptake values of the tumor A maximum
standardized uptake value of> 3.6 was found to be associated with significantly reduced disease-free
survival. Treatment A wide and deep surgical excision with radiation and/or chemotherapy for high-
grade tumors should be performed. Liposarcoma is the most radiosensitive sarcoma (32 % 5-year
survival) Use of chemotherapy. however, remains experimental.

TEACHING ATLAS OF ABDOMINAL IMAGING 348 Prognosis The well-differentiated type and most myxoid
types have favorable prognoses, with 100 and 88% 3-year survival rates, respectively They do. however,
tend to recur even if completely excised Poorly differentiated types have a poor prognosis, with a 5-year
survival rate of -50% PEARL The amount of lipomatous tissue in the tumor can help differentiate
between well-differentiated and higher grade liposarcomas PITFALL Liposarcomas that are a mixture of
fat and soft tissue can appear to be water density on CT due to volume averaging. Suggested Readings
Pereira JM. Sitlin CB, Pinto PS, Casola G. CT and MR imaging of extrahepatic fatty masses of the abdo-
men and pelvis: techniques. diagnosis, differential diagnosis, and pitfalls. Radiographics 2005:25: 69-85
Sung MS, Kang HS, Suh JS, et al. Myxoid liposarcoma: appearance at MR imaging with histologic
correlation. Radiographics 2000:20:1007 1019

*CASE 108 Clinical Presentation A 60-year-old woman presents with abdominal discomfort Fie. 108.1
Axial CT image with oral and intravenous cootrast demonstrates a hary appearance to the mes entery
with small mesenteric ymph nodes Radiologic Findings Axial computed tomography (CT] image with oral
and intravenous contrast (Fig. 108.1) demonstrates a hazy appearance to the mesentery with small
mesenteric lymph nodes Diagnosis Misty mesentery Differential Diagnosis Mesenteric panniculitis
Malignancy, specifically non-Hadgkin lymphoma Inflammation due to processes such as inflammatory
bowel disease and pancreatitis: Infection Mesenteric edema lymphrdema Trauma and hemorrhage
Discussion Background The term misty mesentery refers to increased CT attenuation of the mesenteric
fat caused by infiltra- tion with fluid, inflammatory cells, tumor, or fibrosis. 349

TEACHING ATLAS OF ABDOMINAL IMAGING 350 Clinical findings may be nonspecific or secondary to the
underlying condition. Findings may b dental in an asymptomatic patient imaged for other reasons inci
Clinical Findings Etiology The increased CT attenuation may be the result of infiltration of the mesenteric
fat by a variety of causes depending on the underlying condition. Causes include fluid such as mesenteric
edema due to hypoalbu minemic states, cirrhosis, inferior vena cava obstruction, Budd-Chiari syndrome,
or lymphatic obstruction hemorrhage; or inflammatory cells as in mesenteric panniculitis, tumor cells,
and fibrosis Imaging Findings Misty mesentery refers to nonspecific increased CT attenuation of the
mesenteric fat. Treatment Close follow-up with CT is recommended to search for occult malignancy.
Follow-up examina- tions may demonstrate resolution of the inflammatory condition. In cases of
persistent misty mesentery with unknown cause, biopsy may be necessary for definitive diagnosis.
Prognosis The prognosis depends on the underlying condition PEAR Misty mesentery refers to increased
CT attenuation of the mesenteric fat and is a nonspecific finding that may occur in a variety of clinical
situations PITEALL Failure to recognize misty mesentery on CT may lead to a delay in diagnosis and
treatment of infiltrating neoplastic conditions. Suggested Readings Horton KM, Lawler LP, Fishman EK. CT
findings in sclerosing mesenteritis (panniculitis): spectrum of disease. Radiographics 2003:23:1561-1567
Mindelzun RE, Jeffrey RB, Lane MJ, Silverman PM. The misty mesentery on CT: differential diagnosis AJR
Am J Roentgenol 1996;167:61-65 Seo BK, Ha HK, Kim AY, et al. Segmental misty mesentery: analysis of CT
features and primary causes Radiology 2003:226:86-94 Sheth S. Horton KM, Garland MR. Fishman EX. CT
appearance of primary and secondary tumors and differential diagnosis. Radiographics 2003,23:457-473

*CASE 109 Clinical Presentation A middle-aged woman presents with abdominal pain and vomiting Fig.
109,1 (A-C) Axial and coranal contrastenhanced CT scans show dilation of the small bowel, indicative of a
mechunical ubstruc tion, The obstruction is proximal to the herniated howel loop, seens Mong the
anterolateral pelvic wall between the pectineal and obtu rator externus muscles (orrows) Radiologic
Findings Axial and coronal contrast-enhanced computed tomograplty (CT) scans show dilation of the
stmall bowel, indicative of a mechanical obstruction. The obstruction is proximal to the herniated bowel
loop, seen along the anterolateral pelvic wall between the pectineal and obturator externus muscles
(Fig. 109.1) Diagnosis Small bowel obstruction secondary to obturator hernia 351

TEACHING ATLAS OF ABDOMINAL IMAGINC 352 Differential Diagnosis Other causes of small bowel
ohstruction Discussion Obturator hernias occur when the peritoneal sac and its contents protrude
through the obturator canal. The hernia sac protrudes between the obturator externus and the
pectineus muscle. These her nlas may also be found between the superior and medial fasciculi of the
obturator externus muscle or the external and internal obturator muscles Background Clinical Findings
Obturator hernias are uncommon and difficult to diagnose. They typically occur in elderly, emaciated
women with a history of prior pregnancies and in obese women who have undergone significant weight
loss. Because this hernia is located deep within the abdominal wall musculature, a palpable inguinal
mass is uncommon. Most patients complain of vague groin or abdominal pain, weight loss or recurrent
episodes of small bowel obstruction. Compression of the obturator nerve by the hernia sac produces the
characteristic Howship-Romberg sign, in which compression of the obturator nerve results in pain along
the ipsilateral medial thigh to the knee worsened by adduction, extension, or internal rotation of the
thigh. On physical examination, the hernia sac may present as a tender palpable mass if incarcerated.
Obturator hernias are 6 times more common in women than men and are more often found on the right
side. Other predisposing factors are chronic cough, chronic constipation, and ascites Complications
Because these hernias are difficult to diagnose, appropriate surgical treatment is often delayed, and
incarcerated bowel is common. Early diagnosis is important because of the advanced age and often
debilitated condition of the affected population, with significant morbidity and mortality associated with
this type of hernia Etiology This type of condition is an acquired defect in the obturator membrane that
results in herniationn of the peritoneal sac and its contents through the obturator canal. Imaging
Findings . Plain or barium-enhanced radiographs show a complete or partial small bowel obstruction and
may demonstrate a fixed loop of bowel containing gas or contrast adjacent to the abturator canal or
superior pubic ramus .CT Scan shows the presence of a hernia sac protruding through the obturator
foramen with exten- sion between the pectineal and obturator muscles These hernias most often
contain a loop of ileum but may also contain other bowel loops, omen- tum, bladder, appendix, uterus,
or adnexal tissue

353 VII GI TRACT, PERITONEAL CAVITY AND RETROPERITONEUM Treatment Prompt surgical treatment is
indicated to avoid bowel incarceration. The majority of patients require resection of strangulated small
bowel. Prognosis Obturator hernias have the highest reported mortality rate of all abdominal wall
hernias, ranging from 12 to 70 PEARLS Obturator hernias are most commonly found in multiparous,
emaciated elderly women or in obese women with a significant weight loss Obturator hernias should be
considered in all elderly, debilitated women with small bowel obstructions and no history of prior
abdominal surgery The hernia sac most commonly protrudes between the obturator externus and the
pectineus muscle The Howship-Romberg sign occurs because of compression of the obturator nerve by
the hernia sac, resulting in pain along the ipsilateral medial thigh to the knee worsened by adduction,
exten- sion, or internal rotation of the thigh PITFALLS The signs and symptoms of an obturator hernia are
nonspecific, resulting in delayed preoperative diagnosis Obturator hernias have the highest mortality
rate of all abdominal wall hernias because of delayed diagnosis and resultant incarcerated small bowel.
Suggested Readings Aguirre DA, Santosa AC, Casola G, Sirlin CB. Abdominal wall hemias: imaging
features, complications and diagnostic pitfalls at multi-detector row CT. Radiographics 2005:25:1501-
1520 Harrison LA, Keesling CA, Martin NL, Lee KR, Wetzel LH. Abdominal wall hernias: review of herniog-
raphy and correlation with cross-sectional imaging, Radiographics 1995:15:315-332 Miller PA, Mezwa
DG, Feczko Pl, Jafri ZH, Madrazo BL Imaging of abdominal hernias. Radiographics 1995:15:333-347
*CASE 110 Clinical Presentation A 48-year-old man complains of abdominal pain, severe bladder
pressure, and pain with coughing for 2 years D Fig. 110.1 (A-D) Axial and sagtal images of the abdomen
and pelvis show a large, low-attenuating mass in the right lower quadrant of the abdomen extending inta
the pelvis with intermal septations Radiologic Findings There is a large, low-attenuating mass in the right
lower quadrant of the abdomen extending into the pelvis with internal septations (Fig, 110.1). Diagnosis
Multilocular mesothelial (peritoneal) inclusion cyst 354

355 CI TRACT, PERITONEAL CAVITTY, AND RETROPERITONEUM v Differential Diagnosis Lymphangioma .


Ovarian cystadenoma/cystadenocarcinoma . Cystic carcinoma . Mesenteric-omental cyst Visceral cysts
Discussion Background Multiloculated mesothelial (peritoneal) inclusion vstsor multicyitic mesothelioma
of the peritoneum, s a nonneoplastic proliferation of mesothelial cells lining the peritoneum, omentum,
and abdominal and pevie viscera with a predilection for surfaces of the pelvis. They usuallv occur in
young to middle-aged Wormen. Clinical Findings Most patients are premenopausal women who present
with pelvic abdominal pain Etiology Multiloculated mesothelial (peritoneal) inclusion eysts usually
develop in the setting of fluid secre- tion from active ovaries. The formation of cysts is facilitated by the
presence of peritoneal adhesions, which accumulate fluid and cause reactive proliferation of mesothelial
cells Imaging Findings Cross-sectional imaging demonstrates multilocular thin-walled cysts containing
watery secre- tions, Internal septations may show enhancement on contrast-enhanced computed
tromography or magnetic resonance imaging Treatment Treatment is achieved by total surgical excision
Oral contraceptive treatment may be useful in suppressing fluid secretion from the ovaries Radiation and
chemotherapy are not effective. In rare cases, laparoscopic or open surgical excision of adhesions may be
necessary Prognosis With surgical excision, the rate of recurrence is 30 to 50% PEAR Multilocular
mesothelial (peritoneal) inclusion cysts are forned by the accumulation of peritoneal fluid inside
peritoneal adhesions, which usually occur in premenopausal women. They may mimic cystic neoplasms,
especially those of the ovaries. Preoperative diagnOsis should be considered in the appropriate clinical
setting

*CASE 111 Clinical Presentation An 84-yeat-old woman presents with diarrhea and leukocytosis Fig.
111.1 (A-D) Axial and coronal images from a contrast-enhanced Ct'show an enlarged appendix filled with
soft tissie density material and concentric calcifications There is no significant soft tissue stranding in the
surrounding mesentery, Additionally, there are coarse, rounded calcifications within the lymph nodes
adjacent to the appendix ) Radiologic Findings Axial and coronal images from a contrast enhanced
computed tomography (CT) scan show an enlarged appendix filled with soft tissue density material and
concentric calcifications. There is no significant soft tissue stranding in the surrounding mesentery.
Additionally, there are coarse, rounded calcifica- tions within the lymph nodes adjacent to the appendix
(Fig. 111.1) Diagnosis Porcelain appendix secondary to an appendiceal mucocele 357

TEACHING ATLAS OF ABDOMINAL IMAGING 358 Differential Diagnosis Mucocele of the appendix
Nonneoplastic occlusion of the appendix (e.g. appendicolith Benign neoplasm (eg, mucinous
cystadenoma) Malignant neoplasm (eg, mucinous cystadenocarcinoma) Chronic tubercular or pyogenic
abscess Porcelain gallbladder Degenerated leiomyoma Discussion Background Epithelial neoplasms of
the appendix are less common than carcinoid tumors. Most epithelial neo- plasms occur in middle-aged
or older adults and tend to be mucinous rather than nonmucinous unlike the epithelial tumors
elsewhere in the colon, The term mucocele refers to an appendix that has filled with mucinous material
either due to an obstruction from a nonneoplastic cause or a benign or malignant mucinous tumor of
the appendix. When curvilinear calcifications form around the mucocele, it is referred to as a porcelain
appendix. Clinical Findings Appendiceal mucoceles from benign neoplasms are generally asymptomatic
and may be found inci- dentally on physical examination as a palpable mass or by an imaging study.
Symptomatic patients may present with acute appendicitis, intussusception, torsion, or right ureteral
obstruction. Complications Appendiceal mucoceles may become superinfected and present with
appendicitis. Rupture of a benign or malignant mucocele results in accumulation of gelatinous material
in the peritoneal cavity and is called pseudomyxoma peritonei. In the case of a benign mucocele, only a
localized collection develops However, rupture of a malignant mucocele results in diffuse
pseudomyxoma peritonei with prolifera- tion of malignant celis and spread of gelatinous material
throughout the peritoneal cavity Imaging Findings The diagnosis of a porcelain appendix may be
suggested by an abdominal radiograph when a soft tissue mass with curvilinear calcifications is seen in
the right lower quadrant. A contrast enema will show a smooth impression on the medial aspect of the
cecum . CT scans may be useful in identifying the cause of the porcelain appendix as well as in assessing
for complications, such as superinfection and the development of pseudomyxoma peritonei Treatment
Treatment is surgical excision to eliminate malignant potential and prevent complications, Because of the
risk of rupture and peritoneal extension, a laparoscopic approach is not recommended. Appendectomy
is sufficient for cystadenomas when the appendiceal base is intact, but cecal resec- tion is recommended
for cystadenomas with a large base Cystadenocarcinomas require a hemicolectomy.

359 VIl GI TRACT, PERITONEAL CAVITY AND RETROPERITONEUM Prognosis Patients with a simple or
benign neoplastic mucocele have a 5-year survival rate of 91 to 100% However, patients with diffuse
pseudomysoma peritonei have an unrelenting course with d 5-year survival rate of only -65% PEARL
When curvilinear calcifications are seen in the region of the appendix on an abdominal radio- graph. a
porcelain appendix should he considered. PITFALL Excessive manipulation of the appendiceal mucocele
should be avoided during colonoscopy or surgery to prevent rupture and development of
pseudomyxoma peritonel Suggested Readings Persaud T. Swan N. Torreggiani WC. Giant mucinous
cystadenoma of the appendix. Radiographics 2007:27:553-557 Pickhardt P, Levy AD, Rohrmann CA Jr.
Kende AL Primary neoplasms of the appendix: radiologic spectrum of disease with pathologic
correlation. Radiographics 2003:23:645-662

*CASE 112 Clinical Presentation A 55-year-old woman presents with a 2-month history of abdominal
discomfort, anorexia, and frequent bowel movements after being treated for rectal carcinoma Fig. 112.1
(A,8) Axial images from a contrast CT scan show thicken- Ing pelvic small bowel loops with loss of
nommal folds. (C) Small bowe barium series demonstrates deep ulcers in the small bowel, mucosal
edema, and strictures Radiologic Findings Axial images from a contrast-enhanced computed tomography
(CT) scan show thickening pelvic small bowel loops with loss of notmal folds. A small bowel barium series
demonstrates deep ulcers in the small bowel, mucosal edema. and strictures (Fig 112.1) Diagnosis
Radiation enteritis Differential Diagnosis Bacterial enteritis Inflammatory bowel disease 360

361 VII GI TRACT, PERITONEAL CAVITY, AND RETROPERITONEUM Discussion Background Portions of the
gastrointestinal tract that are in the field of external beam radiation can be affected acutely or
chronically. The small bowel is the most radiosensitive abdominal organ. Within the small bowel, the
ileum is the most commonly affected portion, as it is the least mobile segment. Factors affecting the
development of radiation enteritis include small body size, prior surgery or inflamma tory conditions
causing adhesions, hypertension, diabetes, and combination chemotherapy Clinical Findings Hyperemia,
edema, and inflammation of the submucesa are seen acutely, The subacute phase, detined as 2 to 12
months postradiation, demonstrates partial tissue regeneration and also arteriolar oblitera- tion,
Progressive fibrosis of the bowel will marks the late stage, which can also include adhesion and fistula
formation, The development of radiation enteritis is related to the radiation dose Complications Diarrhea
Malabsorption .Abnormal peristalsis Etiology Related to radiation treatment of adjacent organs Imaging
Findings Barium studies performed during the acute and subac nodular filling defects Chronic-stage
radiation enteritis can show wall thickening, strictures, adhesionis and fistulas stages can show
thumbprinting and Treatment Typically, medical treatment of diarrheal symptoms and dietary
adjustment suffice. Up to 10% of patients who have had 5000 rad of tadiation require surgical
intervention to treat the complications of radiation enteritis Prognosis Acute radiation enteritis
symptoms usually resolve over 2 to 3 weeks, whereas chronic radiation enteritis symptoms may persist
PEARI History of prior radiation, with distinct demarcation of abnormality (edge of the radiation fleld

TEACHING ATLAS OF ABDOMINAL IMAGING 362 PITFALL May be difficult to differentiate from
inflammatory bowel disease and ischemic colitis. Suggested Readings Capps GW, Fulcher AS, Szucs RA,
Turner MA. Imaging findings of radiation-induced changes in the abdomen. Radiographics 1997:17:1455-
1473 Brant WE, Helms CA. The Brant and Helms Solution: Fundamentals of Diagnostic Radiology. 3rd ed
Philadelphia: Lippincott Williams & Wilkins; 1990

*CASE 113 Clinical Presentation A 35-year-old woman with a past history of Hodgkin lymphoma presents
with right-sided abdomi nal pain 856 L820 Fia. 113.1 (A) A noncontrast CT scan shows a large
asymmetrically durmbbell-shapedt mass arising from a right neural foramen at a superior lambar level
The mass measures soft tissue attenuation and caunes widening of the right neural foramen (8-E) On MR
the mass is homogeneously hypointense to muscle on 11, predominantly hyperintense on 12. and dem-
onstrates intense, heterogenedus enhancement after the administration of gadolinium. Coronal imaging
confirms and superiorly displaces the right lidney feontinued) the mass arises from a neural foramen
Radiologic Findings A noncontrast computed tomography (CT) scan shows a large, asymmetrically
dumbbell-shaped mass arising from a right neural foramen at a superior lumbar level. The mass
measures soft tissue attenua- tion and causes widening of the right neural foramen. On magnetic
resonance imaging (MRI) the mass is homogeneousty hypointense to muscle on T1, predominantly
hyperintense on T2, and demonstrates intense, heterogeneous enhancememt after the administration of
gadolinium. Coronal imaging confirms that the mass arises from a neural foramen and superiorly
displaces the right kidney (Fig. 113.1) Diagnosis Retroperitoneal schwannoma (also known as a
neurilemoma or neurinoma) Differential Diagnosis Neurofibroma 363

TEACHING ATLAS OF ARDOMINALIMACING 364 1131 L 559 048 L 673 Fig 113.1 ontnun Lateral
meningucele Paraspinal abscess Sympathetic ganglion tumor (ganglioneuroma) Metastasis or recurrence
of lymphoma Soft tissue sarcoma Discussion Background Schwannomas are benugn tumors arising from
peripheral nerves. They may occur anywhere in the body but are most often found along spinal or
sympathetic nerve roots in the head or neck and flexor surfaces of the upper and lower extremities. The
posterior mediastinum and retroperitoneum are other common locations Schwannomas are usually 5
cm in size: however, retroperitoneal schwannomas are often larger at presentation, as they grow slowly
over d long period of time before detection. The majority of spinal nerve root schwannomas are
intradural extramedullary lesions and are most commonly found in the thoracic spine Clinical Findings
Schwannomas occur in young to middle-aged adults. Most tumors are asymptomatic and may be found
incidentally. They are usually solitary, slow-growing entities with benign features. The presence of
multiple schwannomas may be seen with neurofibromatosis 2 (NF2) Schwannomatosis is a condition in
which patients demonstrate multiple schwannomas without additional clinical features of NF2
Complications Large spinal nerve root schwannomas may cause neurologic symptoms, such as radicular
pain and parasthesias Etiology A schwannoma is a benign neurogenic tumor arising from peripheral
nerve sheaths

VI GI TRACT, PETONE AL CAVITY AND RETROPEITONEUM365 Imaging Findings On CT imaging,


schhwannomas are well-circumscribed, rounded masses associated with a periph eral nerve Thry may
demonstrate cystic degeneration, hemorrhage, or leis commanly, calcifica Bon in spinal nerve root
schwannomas, the neural foramen and interpedicular distance may be widened and the pedicle
Thinned. Large lesions may be seen to expamd the cenral canal MRI characteristics of schwannomas
depend on the type of tissue that they contain Schwannomas containing myxoid tissue will appear
hyperintense on 12 whereas those containing cellular tissue will appear hypointeme on both T1- and 12-
weighted images Those schwannomas with solid fibrous tissue will enhance on contrast-enhanced CT
and MRI Inadequate blood supply to the center of a schwannoma will result in degeneration, resulting in
the target sign, in which lesions demonstrate a T2 hypointense center with a hyperintense rim The
fascicular sign also may be seen, reflecting the presence of fascicular bundles within a peripheral nerve
sheath tumor Treatment Complete surygical resection of the schwannoma and the tnvolved peripheral
nerve root is recom mended Prognosis Schwannomas are slow-growing tumors, and mallignant
degeneration is uncommon Recurrence is rare in patients with solitary lesipns: however, new lesions may
develop in patients with NI2 PLARLS A well-circumscribed, dumbbell-shaped mass that expands
enhancement is.highly stuspicious for a schwannoma The majority of spinal schwannomas are solitary
lesions. Multiple lesions occur with NF2. neural foramen and demonstrates PITFALLS If NF2 is suspected,
the entire neuraxis should be scanned to Tule out the presence of additional lesions It is not possible to
distinguish a schwannoma from a neurofibroma by imaging characteristics Suggested Readings Beaman
FD, Kransdort MJ. Menke DM. Schwannoma: radiologic-pathologic correlation. Radiographics
2004:24:1477-1481 Nishino M. Havakawa K, Minami M. Yamamoto A, Ueda H, Takasu K. Primary
retroperitoneal ne0 plasms: CT and MR imaging findings with anatomic and pathologic diagnostic clues,
Radingcaphics 2003:23:45-57 Rha SE, Byun JY, Jung SE, Chun HJ, Lee HG, Lee JM and imaging
characteristics. Radiographics 2003:23-2 enic tumors in the abdomen: tumor types

*CASE 114 Clinical Presentation An 80-year old man presents wuth abdominal pain, idistention. and
vomiting Fig. 114.1 (A) Asial and (B) coranal CT images show multiple dilated loops of the small
bowrlljejunum iled with contrast There is an oval shaped mised demity bezcar in à loop of distal jqu num
(arows) distal to which the leops of the small bowel are decumpressed Radiologic Findings Axial and
coconal computed tomography (CT) images show multiple dilated loops of the small bowel jejumum
filled with contrast. There is an oval-shaped mixed density lesion in a loop of distal jejunum to which the
loops of the small bowel are decompressed (Fig. 114.1). Diagnosis Mechanical small bowel obstruction
secondáry to phytobezoar Differential Diagnosis Other foreign bodies, such as ingested packets of itlegal
drugs or gallstones 366

36/ v GI TRACT PERITONEAL CAVITY, AND RETROPERITONEUM Discussion Background Dezoars of the
small howel are uncommon, but they usiually result from ingestion of a significant amount of high-fiber
vegetables or fruits. tn recent years, there has been an increased incidence of phytobezaars in patients
who have undergone gastric outlet surgery to treat gastroduodenal peptic ulcer disease Clinical Findings
Patients often present with abdominal pain, nausea, and yomiting, Complete bowel obstruction af the
level of the jejunum or proximal ileum is the most common clinical presentatidn. Complications
Perforation and abscess formation Imaging Findings CT findings include an intraluminal mass outlined by
fluid contained in the small bowel. There is a heterogeneous and mottled appearance of the mass
secondary to pockets of air contained in the interstices. This may appear similar to small bowel
fecalization seen in cases of severe small bowel abstruction Treatment and Prognosis Acute, complete,
and high-grade obstruction requires immediate surgery. Partial obstruction can be managed
conservatively. Strangulation constitutes an emergency, given the risk of ischemia. Suggested Readings
Boudiaf M, Soyer P, Terem C, Pelage IP, Maissiat E, Rymer R. CT evaluation of small bowel obstruction.
Radiographics 2001:21:613-624 Furukawa A, Yamasaki M. Furuichi K. et al, Helical CT in the diagnosis of
small bowel obstruction Radiographics 2001:21:341-355

*CASE 115 Clinical Presentation A'30-year-old man presents with a 1-day history of emesis and diarthea
Fig. 115.1 (A-C) Aial, (D,E) coronal, and (F.G) sagittal images of the abdomen and pelvis show a tubular
strue ture arising from the distal small bowel (arrows) filled with Tas and particulate material. There n
accompanying solt tissue stranding seen surrounding this structure Radiologic Findings Axial, coronal,
and sagittal images of the abdomen and pelvis show a tubular structure arising from the distal small
bowel filled with gas and particulate material. There is accompanying soft tissue stranding seen
surrounding this structure (Fig. 115.1) Diagnosis Perforated Meckel diverticulum Differential Diagnosis
Acute appendicitis Crohn disease leal diverticulitis 368

VCI TRACT, PERITONEAL CAVITY, AND RETROPERITONEUM 369 Fig. 115.1 (continued Foreign body
perforation Perforated neoplasm Discussion Background Meckel diverticulum is the most common
congenital abnormality of the gastrointestinal tract seen with a slight male predominance. The rule of 2s
in Meckel diverticulum is 2% prevalence, 2 feet from the ileocolic valve, and 2 inches in length

TEACHNG ATLAS OF ABDOMNAL IMACING 370 Pabents who present with Meckel diverticulitn ae most
often clinically sunpected of aving acute appendicitis and may present with symptoms suppestive of
acute apperdictis Clinical Findings Complications The total lifetime complication rate af a Meckel
diverticulum - Complications include Bowel obstruction (35x) Especially in adults Secondary to
omphalomesenteric band, internal herma, or vohvufus through or aund vitel- line duct remnants
Hemorrhage (325) Especially in children Secundary to heterotopic gastric mucosa Diverticulitis (223)
Umbilical fistula (10%) Littre hernia: Meckel diverticulum entrapped in an inguinal hernia Neoplasims
Etiology A Meckel diverticulum represents a true diverticulum that contains all three layers of the
intestinal Wall and arises from the antimesenteric border of the small bowel. This diverticulum forms as
a result of incomplete closure af the intestinal end of the omplhalomesenteric duct, which connects the
primitive midgut to the yolk sac during embryonic development. Imaging Findings Patients with Meckel
diverticulum may be imaged with nuclear medicine techniques as well as by computed tomography (CT)
scans Nuclear medicine pertechnetate "Meckel's" scan is an accurate technique that yields positive
results only in the presence of ectopie gastric mucosa and has an overall diagnostic accuracy of 90% in
these patients. This test is most useful in children and has a positive predictive value of only 60% in the
adult population A Meckel diverticulum may be shown at small bowel fol low-through examination:
however, sensitivity for visualizing the diverticulum is increased by performing enteroclysis The CT
appearance of the inflamed diverticulum varies considerably. The diverticulum may appear as a blind-
ending pouch that contains fluid and air or particulate material with absence of oral contrast material.
The inflamed diverticula may also show mural enhancement Treatment .Definitive treatment of à
complication, such as a bleeding Meckel diverticulum, is the excision of the diverticulum aloog with the
adjacent ileal segment

vil CITRACT, PERITONEAL CAVITY AND RETROPERITONEUM 37 Prognosis Generally good if promptly
identified and treated PEARL Tubular structure arising from distal small bowel: suspect Meckel
diverticulum PITFALL May be difficult to identify if not inflamed Suggested Readings Gardner-Thorpe J.
Hardwick RH, Carroll NR Gibbs P. lamieson NV, Praseedom RK. Adult duodenal intussusception associated
with congenital malrotation. World J Gastroenterol 2007:13:3892-5899 Wasike R. Saidi H. Perforated
Meckel's diverticutitis presenting as a mesenteric abscess: case report East Afr Med J 2006:83:580-584
*CASE 116 Clinical Presentation A reenage boy presents with recurrent upper abdominal pain for several
days after beginning new ekercise routine Fig. 116.1 (A,B) Contrast-rahanced avial CT images
demonstrate a wellcircumscribed fatty lesion anterior to the stomach tem) The lesion shows central fat
density with intemal wisps of soft lssoe density and surrounding inflammatory soft tissue stranding
Radiologic Findings Contrast-enhanced axial computed tomography (CT) images demonstrate a well-
circumscribed fatty lesion anterior to the stomach. The lesion shows central fat density with internal
wisps of soft tis- se density and surrounding inflammatary soft tissue stranding The absence of adjacent
gastric wall thickening is a pertinent negative finding (Fig. 116.1) Diagnosis Focal omental infarction
Differential Diagnosis Mesenteric lipodystrophy (a subset of sclerosing mesenteritis) Acute epiplaic
appendagitis (EA) Acute diverticulitis Fat necrosis associated with pancreatitis Lipasarcoma 372

aRACT PRITONEAL CATY AND ETROPERITONEUM 373 Discussion Background The greater omentum
which composed of a double layer of periteneum oipnates from the gmater curvature of the stomach
and extends anteromferiorly to drape over the raoese colon. I has a variable length of 15 to 36 cm
Omental infarct actually represents osly a segmental infarcrion of the greateromentum infarCtion of the
entire omentum has not been reported Following an insult the classic pathologic findings of infarction
develop, including vascular congesion or thrombosis ssue necrosis, inlammatory cell aggresation, and
hemonhage The majority of omental intarction Ncurs in the right aspect of the omentum Clinical
Findings Approximately 85% of cases occur in adults and 15% in the pediatric population Most patiemts
present with acute abdominal pain. Classically, there is focal tenderness, and the pain localizes to the
nght abdomen (upper or lower quadrant Because of this localization, the presentation can mimic appen
dicitis, ulcer disease, or cholecystitis, Elevation of the white blood cell count or fever is unusual
Complications Serious complications of omental infarct are rare. However, the inflammatory response
may heal with resultant intraperitoneal adhesions, which can secondarily cause bowel obstructions
Reactive ascites is not common in omental infarction: however, superinfection and abscess formation
have been reported Etiology Omental infarct likely represents the sequelae of multiple etiologies. These
etiologies lead to decreased perfusion to the omentum. Physical torsion r direct trauma of the omentum
can cause stenosis o kinking of the arterial supply or venous drainage of the omentum. Anomalous or
congenitally stenosed arterial supply has also been suggested. Kinking of the veins has also been
hypothesized due to increased intra-abdominal pressure. The most frequent etiology may be thrombosis
of the omental draining veins whether from a hypercoagulable state or direct injury with resultant
venous conges- tion and infarct. Predisposing clinical factors include abdominal trauma or surgery,
obesity, strenuous activity (a form of trauma), congestive heart failure, and digitalis use Imaging Findings
Acute nonspecific and focal right abdominal pain is overwhelmingly evaluated with CT or, in the pediatric
age group, also with altrasound. The omental infarction appears as a fatty lesion in the anterior
peritoneum. As in the case depicted. it has both internal and surmrounding soft tissue density
inflammatory stranding. Because the omentum arises from the stomach and is intimately

TEACHING ATLAS OF ABDOMINAL IMAGING 374 associated with the peritoneal portions of the colon, the
infarct often appears to be involving the serosa of the colon or stomach Although reactive thickening of
the adjacent bowel has been reported, it is arypical If the omental inlarct occurs in a region of colon
containing a diverticulum definitive differentiation of these two entities by imaging may not be possible
Diverticulitis is statistically more common in the descending and sigmoid colon, whereas omental infarcts
are traditionally seen anterior to the transverse and ascending colon. EA also presents as an
inflammatory fatty mass intimately associated with the colonic wall. The characteristic central
hyperdense focus of EA may be a helpful discriminating factor, as it is not seen with infarcts. Omental
infarcts can be quite large in size, 10 cm or more in maximal diameter whereas it is unusual for EA to
present with > 5 cm of inflammatory change. A true distinction between EA and omental infarct may not
be clinically relevant because management is with expectant pain treatment for both Treatment The
mainstay is symptomatic treatment of the patient's pain, usually with nonsteroidal anti- inflammatory
medications. Antibiotics are not routinely administered. Some pediatric surgeons advocate laparoscopic
resection of the inflammatory mass for quicker pain relief, although this recommendation is not
universal. If as mentioned above, the omental infarct occurs in the location of adjacent diverticulosis and
dif ferentiation from diverticulitis is not possible, appropriate antibiotic coverage may be indicated
Prognosis The complications listed above are generally rare: therefore, prognosis for this benign entity is
excellent, Most patients symptoms resolve without further problems or recurrence Dedicated follow up
imaging is not needed when the initial clinical and imaging presentation is classic If imaging is done for
other reasons, the infarct may show healing with a small resultant mixed fatty-fibrous focus and/or
dystrophic calcification PEARLS AND PITFALLS Focal omental infarction is a benign process that, on
clinical presentation, can mimic acute appen- dicitis, cholecystitis, or diverticulitis CT imaging is usually
diagnostic and obviates surgical intervention, as complications are rare When the omental infarct occurs
in the setting of diverticulosis, antibiotic therapy and follow-up imaging may be prudent to avoid false-
negative diagnosis of diverticulitis, a mimic of omental infarction.
*CASE 117 Clinical Presentation complains of abdominal swelling A 30-year-old woman D Fig. 117.1 (A)
Contrast-enhanced axial image shows a moderately partment of the anterior abdominal wall. (B)
Noncontrast axial T1-welghted image shows a well-defined midline abdominal wall lesion isointense to
muncles in the muscular compartment. (C) Axial 12-weighted defined midline anterior abdominal wall
lesion isointense to muscles. (D) Contrast-enhanced coronal T1-weighted image shows an avidly
enhancing, well-delined, midline anterior abdominal wall lesion. enhancing homogeneous lesion in the
muscular com anterion image shows a well- Radiologic Postcontrast computed (MRI) show a well-
defined lature in the midline that is isodense and isointense to muscles (Fig. 117.1). Findings tomography
(CT) scan and pre- and postcontrast homogeneously magnetic resonance imaging enhancing lesion in the
anterior abdominal wall muscu- 376

VII CI TRACT, PERITONEAL CAVITY AND RETROPERITONEUM 377 Diagnosis of Anterior abdominal wall
desmoid tumor (aggressive fibromatosis) Differential Diagnosis of Malignant soft tissue sarcoma Benign
myositis ossificans Arteriovenous malformation. Extranodal lymphoma Fibrosarcoma Malignant
Metastatic fibrous histiocytoma deposits (usually lung carcinoma) Discussion Background Desmoid
tumor is a benign neoplasm, composed of spindle cells, that arises in musculotendinous musculature
most commonly). It commonly affects metastasize to distant organs, which is why tumors occur in
association with familial adenomatous tumors occur with gastrointestinal polyps, which are
premalignant multiple osteomas, epidermoid cysts, and others so frequently in the areas of prior
abdominal tumors tend to be more aggressive than superficial ones and grow tissues (anterior
abdominal wall and shoulder postpartum women, is locally it is termed benign. In children, desmoid
polyposis (FAP) syndrome in 10 to 15% of patients. In Gardner syndrome, aggressive dismoid, and does
not tissue tumors Desmoid tumors in association with FAP Occur more surgery Intra-larger in size by the
time of clinical presentation Clinical Findings A painless lump in the anterior abdominal wall is the classic
presenting symptom in cases of abdominal wall desmoids. presentation of an intra- of invasion of the
surrounding viscera and rectal bleeding-abdominal desmoid depending on the degree Presenting
symptom oms include changes in bowel habits, pain, Complications The invasion of surrounding organs
as a result of the locally aggressive leads to intestinal obstruction, neurovascular deficit, and bleeding.
The invasion of surrounding neu- tovascular structures also makes surgical resection of the tumor nature
of desmoid tumors more difficult, Etiology The etiology of desmoid tumors is unknown. Possible causes
include surgical trauma and factors (because they are more commonly menopausal and with oral
contraceptive use). endocrine seen in postpartum women and tend to regress during

378 TEACHING ATLAS OF ABDOMINAL IMACING Imaging Findings lobulated lesion in the muscle plane
shows a homogeneous hypoechoic, rounded covering it Ultrasound CI shows a homogeneously
enhancing mass iso- to hypodense to the muscle MRI shows a homogeneous avidly enhancing iso- to
hyperintense and hyperintense and heterogeneous on T2-weighted Hypointense bands seen on all
sequences are characteristic of desmoid tumors elevating the fascia mass on T1-weighted images
recovery images and short T1 inversion Treatment normal margins. Radio- The treatment of choice for a
desmoid tumor is surgical excision with therapy, tamoxifen, and interferon-u are other treatment
options; they are thought to reduce the rate of recurrence and are important in cases with neurovascular
invasion by the rumor Prognosis The recurrence rate tumors reach a "plateau phase." after which they
cease to grow, In such therapeutic approach is advisable. higher in young patients. Desmoid cases, a less
aggressive after surgical resection is 25 to 65%, PEARL The location of the lesion on MRI is suggestive of
this diagnosis PITFALL Although MRI findings are characteristic, they are not specific for desmoid tumor.
Biopsy confir- mation is frequently necessary for confirmation of the diagnosis Suggested Readings
Desmoid Tumor Research Foundation Web site. Available at: http://www.dtrf.org. Accessed September
2, 2007 Ferenc T, Sygut J. Kopczynski J, et al Aggressive fibromatosis (desmoid tumors): definition,
occurrence pathology, diagnostic problems, clinical behavior, genetic Mitchell G. Thomas JM, Harmer CL.
Aggressive fibromatosis 1998 2:149-154 background. PolJ Pathol 2006:57(1):5-15 evidence for a stable
phase. Sarcoma Lee JC, Thomas JM, Phillips S, Fisher C. Moskovic E Aggressive fibromatosis: MRI logic
correlation. AJR Am 1 Roentgenol 2006:186(1):247-254 features with patho-

*CASE 118 Clinical Presentation A 73-year-old woman presents with abdominal discomfort and flank
pain 118.1 Coronal reformatted contrast-enhanced CT Image shows a heterogeneous, ill-defined mass in
the left perinephric reglon surrounding the left kidney. The lesion shows areas of fat density and foci of
calcifications adjacent to the left kidney Fig Radiologic Findings Coronal reformatted contrast-enhanced
computed ill-defined mass in the left perinephric region surrounding the left kidney. The lesion shows
areas of fat density and foci of calcifications adjacent to the left kidney (Fig. 118.1). tomography (CT)
image shows a heterogeneous Diagnosis Dedifferentiated retroperitoneal liposarcoma Differential
Diagnosis Malignant fibrous histiocytonma Retroperitoneal lymphoma Metastases Retroperitoneal
dermoid Infection (rare) . Discussion Background Linosarcoma is the second most common type of soft
tissiue sarcoma in adults and is most commonly seen in the 5th decade of life. 379

380 TEACHING ATIAS OF ABDOMINAL IMAGING Clinical Findings Abdominal fullness Flank pain Vomiting
Abnormal renal function tests in patients with renal invasion Hematuria Complications Invasion of
adjacent organs mainly kidneys Metastases (very rare) Etiology Unknown Imaging Findings large, ill-
defined, heterogeneous fat-containing across the fascial planes mass with Seen on imaging is a
retroperitoneal involvement of adjacent organs and bridging large in size at presentation, CT may The
involvement of adjacent be carefully for complete assessment because tumor tends to invade adjacent
organs without any dence of involvement on imaging The presence of fat on CT and magnetic resonance
tion; however, macroscopic fat is not always present in liposarcomas (Fig. Tumors are almost always and
patients usually present with nonspecific symptoms show calcifications that are commonly nodular in
configuration. organs, mesenteric vessels, and renal vessels is common and should organ remains an
important issue direct evi- assessed at imaging Contiguity with an adjacent imaging (MRI) helps in
diagnosing the condi- 118.2), Treatment Surgical resection of the tumor is the treatment of choice in
patients without metastases Contiguity with kidneys necessitates nephrectomy in addition to resection
of the primary neoplasm Fig. 118.2 Axial contrast-enhanced CT image in a differ- ent patient shows a
hypoattenuating enhancing capsule. The lesion is posterior to the kidney. compressing and displacing the
kidney anteriorly. Because no discrete focus of macroscopic fat was seen, it was dif- ficut to diagnose this
lesion as liposarcoma, however, the lesion was found to be dedifferentiated liposarcoma on sun gical
pathology lesion with a thick

V GITRACT, PERITONEAL CAUTY AND RETROPERITONEUM38 Prognosis The prognosis is worse in patients
with renal and/or inferior vena cava However, in patients with adjacent organ involvement, the prognosis
remains excellent after follow-up imaging is important to detect recurrence involvement and vascular
encasement of the aorta surgically resectable liposarcoma withour treatment, althugh long-term PLARL
Large retroperitoneal mass, which is ill-defined, heterogeneous, and has fat density PITYALL Lesions that
do not demonstrate macroscopic fat on MRI and fat densiry ferentiated from other retroperitoneal
sarcomatous lesions, necessitating biopsy on CT cannot be di Suggested Readings Gupta AK. Colhan RH.
Francis IR, Sondak AJR Am J Roentgenol 2000;174:1025-1030 VK, Korobkin M. CT of recurrent
retroperitoneal sarcoma Nishino M, Hayakawa K. Minami M, et al. CT and MR imaging findings with
anatomic and pathologic diagnostic clues. Radiographics 2003:23:45-57

*CASE 119 Clinical Presentation woman presents with vague abdaminal pain A 64-year-old Fig. 119.1 (A)
Axial contrast-enhanced CT image peritoneal fat and stranding passing through an abdominal wall defect
(orows) in the left flank (B.C) There is soft tissue density from fat necrosis (orrow) seen within the
herniated tat shows retro- Radiologic Findings Axial contrast-enhanced computed tomography (CT)
images show passing through an abdominal wall defect in the left the herniated tat (Fig. 119.1).
retroperitoneal fat and stranding density seen within flank, There is soft tissue Diagnosis Lumbar hernia
of Grynfeltt through the superior lumbar triangle Differential Diagnosis Lipoma Inferior lumbar hernia
(Petit hernia) 382

VII GI TRACT, PERITONEAL CAVITY AND RETROPERITONEUM 383 Discussion Background Grynfeltt is a
hernia through the superior hernias may be congenital (20) or acquired (80%) are due to defects in the
musculoskeletal system. Acquired Jumbar hernias can be primary or second- arv. Primary lumbar hernias
(55% of all lumbar hernias) are spontaneous as surgery, infection, or trauma. Risk factors for primary
lumbar hernias are age, extremes of body habitus, quick postoperative Herniation common. (Grynfeltt)
triangle. Inferior and superior Congenital hernias are discovered in infancy and lumbar lumbar without a
causal factor, such weight loss, chronic disease, muscular atrophy. chronic sepsis, and through bronchitis,
wound infection, strenuous physical activity the superior lumbar triangle, also known as the Grynfellt
triangle, is more The superior lumbar triangle is defined by the quadratus lumborum muscle medially,
the 12th nib superiorly, and the internal oblique muscle laterally. The floor of the triangle is the transver-
salis fascia and the aponeurosis of the transversalis muscle of the abdomen. is the external oblique and
latissimus The roof of the triangle dorsi muscles Clinical Findings Lumbar hernias may contain the small
or large bowel, mesentery, omentum, stomach. ovary, spleen. or kidney. Patients with lumbar hernias
can present with a variety of symptoms. including a pos- terolateral mass (if contents are renal). back
pain, bowel obstruction (if contents contain bowel), and urinary obstruction Complications Bowel
herniation and strangulation Abscess formation Herniation of retroperitoneal structures Imaging
Findings Typically present on the left, superion roperitoneal structures or bowel. They terior Stranding of
the fat may requiring surgical lumbar hernias are best seen on CT when containing ret may present as
protrusions of structures through the pas- abdominal wall into the subcutaneous tissues. reflect
strangulation of the blood supply of the herniated structures repair Treatment and Prognosis Lumbar
hernias will gradually enlarge and lead to symptoms ranging from low back pain to bowel strangulation
with significant associated morbidity. About 25% of lumbar hernias have ated bowel. All lumbar hernias
require surgical repair incarcer-

Das könnte Ihnen auch gefallen